You are on page 1of 77

FEM for Engineering Applications

Exercises with Solutions

Jonas Faleskog
KTH Solid Mechanics
August 2008

1. Elastic Energy and Energy principles

2. Matrix formulated structural mechanics—direct method

3. Strong/weak form and FEM-equations

4. FEM: trusses and beams

5. FEM: planar frames of trusses and beams

6. FEM: 2D/3D solids

7. FEM: heat conduction


FEM for Engineering Applications—Exercises with Solutions / August 2008 / J. Faleskog

1. Elastic Energy and Energy principles


1.1 A simply supported beam of length 3L and with θ
bending stiffness EI is subjected to an external
bending moment M0, see the figure to the right.
Determine the rotation θ by use of Castigliano’s L 2L
theorems. Neglect possible contributions from
shear forces when evaluating the complementary M0
elastic energy.

1.2 Two beams, each with bending stiffness EI and


C
length L, are connected at B, see the figure to left. 45o 45o
The left beam is clamped to a rigid wall at A and a P
point force is applied on the right beam at point C. A

L
E,

A,
A,
Determine the rotation of the corner B by use of an

E,
L
energy method (neglect possible contributions
from normal forces and shear forces to the energy B
expression).

1.3 Determine the horizontal displacement at


point B in the beam structure to the right. All EI, L A
beams are of length L and have the bending

EI, L
stiffness EI. Use complementary elastic energy
and Castigliano’s theorems in the analysis.
B P

1.4 A plane structure is composed of four equal beams


as shown to the right. The beams are of the length L and
have the bending stiffness EI. A moment M0 is applied M0
at the centre point of the structure. Use a suitable
energy method based on complementary elastic energy
to determine the rotation of the centre point.

1.5 A cantilever beam of length L and with bending stiffness EI is P


clamped at its left end, and attached to a spring at its right end. The EI, L
3
spring constant is k = η EI ⁄ L . A point force, P, is applied on the k
beam according to the figure to the right. Determine, by use of a
suitable energy method, the deflection of the right end of the beam.

1.6 A beam of length 2L and with bending stiffness EI is


subjected to a bending moment applied at its right end,
k
see the figure to the right. The left end of the beam is L L
clamped, and the mid point is attached to a vertical spring
3 EI M0
with spring constant k = 6EI ⁄ L . Evaluate the rotation
of the right end of the beam by use of an energy method.

– 1.1 (10) –
FEM for Engineering Applications—Exercises with Solutions / August 2008 / J. Faleskog

1.7 A beam with bending stiffness EI and total length 2L, M0


is simply supported at its mid point. The left end of the L L
beam is attached to a linear spring with the spring constant
3
k = ηEI ⁄ L . The beam is subjected to a point force P0 k P0
and a moment M0. Determine M0 such that the deflection
of the right end of the beam becomes zero. Carry out the
analysis using an energy method, based on complementary elastic energy.

1.8 A system of two beams, each with


bending stiffness EI and length L, and a P P
spring kN, see Figure (a) to the right, has (a) (b)

EI, L

EI, L
proven to be to compliant in an application.
The system is therefore made stiffer by use
of a torsion spring kM, see Figure (b). The
EI, L EI, L
complementary elastic energy for the sys- kM
tem is kN kN

3 2 2
L 1 2 M 2 1 ( P – M ⁄ L ) - 1--- -------------------
(M ⁄ L) -
W = ------ --- ⎛⎝ P + ⎛⎝ P – -----⎞⎠ ⎞⎠ + --- ---------------------------- + ,
EI 6 L 2 k L3 ⁄ EI 2 k M L ⁄ EI
N

where M is the moment arising in the torsion spring when the system is loaded by a point force
3
P. The stiffness of the springs can be expressed as k N = α EI ⁄ L and k M = β EI ⁄ L , where α
= 3 in the current application. Determine β, such that the stiffness of the system increases by a
factor of two, i.e. such that the displacement at P in (b) becomes half compared to the case in
(a). Hint: problem (b) is statically indeterminate.

1.9 The constraint and the boundary conditions of a beam


with bending stiffness EI and length 2L is shown in the P
right hand figure. Determine the vertical displacement of
the beam at the point force. Use an energy based method. L L

1.10 A beam with bending stiffness EI and total


Q
length 3L is subjected to a uniformly distributed load
with the resultant Q, see the figure to the right.
Determine all reaction forces acting on the beam. L 2L
Use an energy method, based on the complementary
elastic energy, for statically indeterminate quantities.

P
1.11 A freely supported beam with bending stiff-
ness EI and total length 3L is loaded by a point A B
force, P, according to the right hand figure. Deter-
mine the deflection at point B.

– 1.2 (10) –
FEM for Engineering Applications—Exercises with Solutions / August 2008 / J. Faleskog

A 2L B
1.12 A plane frame is composed of three beams connected
at the stiff joints B and C, see the figure to the right. The D
L
bending stiffness of all beams are EI. The frame is loaded 2L
by a point force. Determine the vertical displacement of C
point C and evaluate the distribution of bending moment in P
the frame.

The examples below are taken from “Exempelsamling i Hållfasthetslära” , Eds. P.-L. Larsson & R.
Lundell, KTH, Stockholm, january 2001. The solutions to these problems (in Swedish) are based on
Castigliano’s theorems.

L
1.13 Determine the horizontal displacement at point B. The M
bending stiffness of each beam in the planar frame is EI. C
B
L
A

1.14 A planar frame constructed by two beams, each


with bending stiffness EI, is loaded by a uniformly
distributed load with the resultant P and a point force L
P according to the right hand figure. Calculate the P P
vertical displacement at the point force.

L L

A D
1.15 A beam with circular cross section (diameter
P
d) is shaped as a U, see the figure to the right. The
beam is clamped at point A and loaded by a point
force P, acting perpendicular to the plane of the
beam, at point D. Calculate the displacement at L d
point D in the direction of the point force.

B C
L

– 1.3 (10) –
FEM for Engineering Applications—Exercises with Solutions / August 2008 / J. Faleskog

1.16 A planar frame, according to the right hand fig- A P


ure, is clamped at point A and point B. The frame is L
loaded perpendicular to its plane by a couple of point B
forces, each of magnitude P. The beam has a circular
tube shaped cross section with mean radius r and L
C
thickness t (assume that t « r ). The material has the
elastic modulus E and the shear modulus G. Calculate P D
the out-of-plane displacement at points B and C. B
L P
Consider the case when L = l. A
l
L
C P
D

P P
1.17 A rectangular planar frame with measurements
according to the figure is clamped at point A and point D. B C
Two point forces is applied perpendicular to the frame in L
opposite directions at points B and C, respectively. The
bending stiffness of the frame is everywhere equal to EI A D
and its torsional stiffness is GK, with EI ⁄ ( GK ) = κ ,
where κ being a non-dimensional constant. Calculate the 2L
displacements perpendicular to the plane at the points B
and C, respectively.

1.18 A planar frame of quadratic shape is freely supported at the


B C
corner points A, B, C and D, such that only reaction forces per-
pendicular to the frame may occur. The side of the frame is of Q
length L. The bending stiffness and the torsional stiffness are EI
and GK, respectively. The frame is subjected to a uniformly dis-
tributed load with the resultant Q acting perpendicular to the L, EI, GK
frame on the side AB. Calculate the displacement at the mid
point between A and B.
A D

– 1.4 (10) –
FEM for Engineering Applications—Exercises with Solutions / August 2008 / J. Faleskog

Solutions
1.1
RB Equilibrium R = M ⁄ ( 3L )
Introduce reac- B 0
tion forces: RA gives:
M0 RA = R B = M ⁄ ( 3L )
M
RB = ------0-
M0 3L
M1 M1 M0 M2 M2
RA = -------
3L
Equilibrium requires: M 1 = M0 ⁄ 3 M 2 = – 2M 0 ⁄ 3
2 2 2
LM 2LM M0L ∂W M0 L
Complementary elastic energy: W = ----------1- + -------------2- = ----------
- ⇒ θ = ---------- = ----------
-
6EI 6EI 6EI ∂M 0 3EI

1.2

Introduce a fictitious bending moment, MF


and reaction forces, R & MR Equilibrium:
M1 = M F + PL ⁄ 2
R P P P
MR MF M2 = PL ⁄ 2
Equilibrium: P P
R=P M1 M2
MF MR = MF
P P
M1 M2
Complementary elastic energy: MF
2 2 2 3 2
MF L M F PL P L- ∂W PL
W = ----------
- + -----------------
- + ----------- Rotation at B: θ B = ----------- = ----------------
2EI 2 2EI 6EI ∂MF 2 2EI
MF = 0

1.3

1 statically indeterminate, chose e.g. MR


MR M0 M0 L 2 2 2
W = --------- ( M R – M R PL + 2P L )
6EI
∂W- PL
---------- = 0 ⇒ M R = -------
∂MR 2
M0
Displacement at point B:
Equilibrium: M 0 = – PL 3
∂W 7 PL
δ = -------- = ------ ---------
P ∂P 12 EI

– 1.5 (10) –
FEM for Engineering Applications—Exercises with Solutions / August 2008 / J. Faleskog

1.4
Cut and use the symmetry M0
V Equilibrium: ------- – M R – VL = 0
properties! 4
M0/4
V M0/4 V => 1 statically indeterminate, e.g. MR

MR M0 Total complementary elastic energy:


V M0/4 V M0/4
M0/4 L 2 M M 2
W = 4 ⋅ --------- ⎛⎝ MR + M R ------0- + ⎛ ------0-⎞ ⎞⎠
V 6EI 4 ⎝ 4⎠

∂W- M ∂W M0L
---------- =0 ⇒ M R = – ------0- ; Rotation: θ = ---------- = -----------
-
∂M R 8 ∂M0 16EI

1.5

P
MR Equilibrium, beam: Total complementary elastic energy:
2
: –V – N + P = 0 L 2 N
V W = Wbeam + Wspring = --------- MR + ------
N : MR – PL + NL = 0 6EI 2k
2
L 2 N
=> 1 statically indeterminate, = --------- ( PL – NL ) + ----------------------3-
EI 6EI 2ηEI ⁄ L
k = η -----3- chose e.g. N
3
L L 3
= --------- ⎛⎝ P + N ⎛⎝ 1 + ---⎞⎠ – 2PN⎞⎠
2 2
6EI η
∂W- ηP
Principle of least work: ------- = 0 ⇒ N = ------------- ;
∂N 3+η 3
∂W PL 1
Deflection at the right end (Castigliano’s 2nd theorem): δ P = -------- = --------- -----------------
∂P EI ( 3 + η )

1.6

6EI
k = --------
3
-
L Equilibrium gives
N
R M0 – M R Note! one statically
N = --------------------
- indeterminate exists!
MR M0 M0 M0 M0 L
Complementary 2
L 2 2 L 2 N L 2 2
elastic enerrgy: W = --------- ( MR + M R M0 + M0 ) + --------- ( 3M0 ) + ------ = --------- ( MR + 3M 0 )
6EI 6EI 2k 4EI
∂W- L
Condition to determine the unknown ---------- = --------- 2M R = 0 ⇒ M R = 0
reaction force: ∂M R 4EI
∂W 3 M 0 L-
Rotation at the point where the external moment is applied : θ = ---------- = --- ----------
∂M 0 2 EI

– 1.6 (10) –
FEM for Engineering Applications—Exercises with Solutions / August 2008 / J. Faleskog

1.7

Free body diagram: R

M0
M1 M2 P0
N P0 N

EI Equilibrium: Equilibrium:
k = η -----3-
L : M 0 – P0 L + NL = 0 M1 = NL = P 0 L – M 0
N = P0 – M0 ⁄ L

M2 = P0 L
Complementary elastic energy:
2 2
M1 L M2 L N2 3
L -⎛ 2 ⎛M 0⎞
2 2P M
0 0⎞
3
L -⎛ 2 ⎛ M 0⎞
2 2P M
0 0⎞
W = ----------
- + ----------- + ------ = -------- 2P + ------
- – ----------------
- + ------------ P + ------
- – ----------------
-
6EI 6EI 2k 6EI ⎝ 0 ⎝ L ⎠ L ⎠ 2ηEI ⎝ 0 ⎝ L ⎠ L ⎠
∂W 3 + 2η
δ P0 = --------- = 0 (given condition) ⇒ M0 = ---------------- P0 L
∂P0 3+η

1.8
3
∂W PL
Case (a): no torsion spring (M = 0), kN = 3EI / L3 ⇒ δa = -------- = ---------
∂P M=0
EI

Case (b): statically indeterminate problem,


where M is an internal indeterminate quantity, thus
3 3
∂W 2β - ∂W L 2M PL 9 + 2 β
-------- = 0 ⇒ M = PL --------------- δ b = -------- = ------ ⎛⎝ P – --- -----⎞⎠ = --------- ----------------
∂M 3 + 2β ∂P EI 3L EI 9 + 6 β
1 9
According to the given conditions: δ b = --- δa ⇒ β = ---
2 2

1.9
2P
P P
Equilib: : M 0 = PL
P M0 M0 M0 P (statically indeterminate probl.)
P P
L
2 2 3
M- ⎧ x⎫ P L
Total complementary elastic energy: W = 2 ∫ -------- dx = ⎨ M( x) = PL --- ⎬ = ------------
2EI ⎩
L⎭ 3EI
0
3
∂W 2 PL
Displacement at the point force (Castigliano’s 2nd theorem): δ P = -------- = --- ---------
∂P 3 EI

– 1.7 (10) –
FEM for Engineering Applications—Exercises with Solutions / August 2008 / J. Faleskog

1.10
RA Q
Equilibrium: RA + RB – Q = 0 Note! one statically
indeterminate, choose
Qx/L M A + RB L – Q2L = 0
MA RB for instance MA.
QL x 2
M ( x ) = -------- ⎛⎝ ---⎞⎠
M(x) 4 L
x
2L M ( x ) 2
L
Complementary elastic energy: W = --------- ( M A + MA QL + ( QL )2 ) + ∫ --------------- dx
2
6EI 0 2EI
∂W QL 3Q 5Q
----------- = 0 ⇒ M A = – -------- ; R A = – ------- ; R B = -------
∂M A 2 2 2

1.11

Introduce a fictitious force Q when the comple- P Q


L L L
mentary elastic energy, W , is calculated.

L 2 2
The comp. elastic energy in the beam becomes: W = ------------ ( 4P + 7PQ + 4Q )
18EI
3
∂W 7- PL
Displacement at B (Castigliano’s 2nd theorem): δB = -------
- = ----- ---------
∂Q Q = 0 18 EI

1.12
2 equilibrium Eqs. and 3 unknown reac-
RA tion forces (RA, RD and MD). Thus, the
RD problem has one statically indetermi-
nate. Treat RA as known when calculat-
MD ing the complementary elastic energy.
P
L 2 2
Complementary elastic energy in the beam: W = --------- ( 76RA – 40RA P + 8P )
6EI
∂W
--------- 5
The unknown RA is given by: = 0 ⇒ RA = ------ P
∂RA 19 3
Displacement in point C (Castigliano’s 2nd theorem): δ C = ∂W -------- = 52
------ PL
---------
∂P 57 EI
Bending moment diagram:
18
10 ------ PL
------ PL 10 19
19 ------ PL
19
10
------ PL
19

– 1.8 (10) –
FEM for Engineering Applications—Exercises with Solutions / August 2008 / J. Faleskog

1.13

1.9

1.14

1.15

1.16

– 1.9 (10) –
FEM for Engineering Applications—Exercises with Solutions / August 2008 / J. Faleskog

1.18
1.17

– 1.10 (10) –
FEM for Engineering Applications—Exercises with Solutions / August 2008 / J. Faleskog

2. Matrix formulated structural mechanics—direct method

k3
2.1 A system of five springs are connected as k1
shown in the figure to the right. All the spring
constants ki are in the current application
k4
equal to k. Furthermore, D1 = D4 = 0, F3 = 0 k5
and F2 = P. Determine the displacements and k2
reaction forces.
D1, F1 D2, F2 D3, F3 D4, F4

4a
2.2 Three springs are connected according
to the figure to the right, also showing the
applied external force P. The spring con- k2
stants are: k1 = 5k, k2 = k and k3 = 2k. k1 3a
Determine the displacements at the points k3
where the springs are connected and evalu- rigid beam
ate all the reaction forces.
P

2.3 Determine the displacement at the


45o 45o
point force in the spring system shown to k1 = k
the right. k2
k1 45o k2 = 2k
k3
k3 = 2k
P

1 2
(−1,1) y/L
(1,1)

2.4 The plane structure to the right consists of four k1 k2


springs with spring constants: k1 = k2 = k4 = 2k and 3
k3 = 4k. The springs are attached to five nodes with x/L (1,0)
coordinates shown in the right hand figure. The struc- P
ture is loaded by two point forces acting in node 4 5 k3
k4
and node 5, as shown in the figure. Calculate the dis-
placement at each node and the normal force acting (1,−1) 4
in spring element number 4.
P

– 2.1 (12) –
FEM for Engineering Applications—Exercises with Solutions / August 2008 / J. Faleskog

2
2.5 A plane truss structure consists of three truss elements con-
nected to four nodes, as shown to the right. All trusses have cross L L/2
sectional area A and elastic modulus E. The length of each truss
3
element is evident by the figure. A point force, P, is acting on node
4. Calculate the displacements at the nodes and the reaction forces
at nodes 1 and 2, respectively. Show also that global equilibrium is L
satisfied in the vertical direction. 4

2.6 The plane frame structure to the right contains two


truss elements and two spring elements. The spring con-

EA,L
stant for both springs is k = ηEA ⁄ L . The truss ele- P
ments are of length L, have cross sectional area A and α
elastic modulus E. The structure is subjected to a point EA, L k φ
force P according to the figure. The displacement will
for the present structure always be in the direction of the
force P. Determine the relation between δ and P. π-
k -- –φ
2

2.7 The plane structure in the figure to the right contains


two truss elements and two spring elements. The truss
30o 30o

L
elements have the same length L, cross sectional area A P
E,

A,
and elastic modulus E. The stiffness of the left spring is
A,

E, ϕ
k 1 = 0.75 EA ⁄ L . A point force P is applied on the struc-
L

ture, acting at an angle ϕ, as shown in the figure. Deter-


mine the stiffness of the right spring, k2, such that the k1 k2
displacement always is aligned with the force P, i.e. in the
direction of the angle ϕ.

2.8 A mass m0 is attached to three similar springs. The


ϕ
division between the springs is 120o and the spring con- x
stants are k1 = k2 = k3 = k. The springs are attached to a y rigid
rigid ring of radius R. The coordinate system shown in the ring
figure is fixed to the ring, where the y-axis is located in k1
the direction of spring k1. The circumferential position of k
m0 3
the ring is determined by the angle ϕ. Calculate the dis-
g
placement of m0 in the x- and y-directions for an arbitrary k2
angle. The acceleration of gravity g (see the figure) is
assumed to be known.
Hint: derive the equation system with reference to the given xy-coordinate system.

– 2.2 (12) –
FEM for Engineering Applications—Exercises with Solutions / August 2008 / J. Faleskog

2.9 An adjustable crane consists of two rods which are connected at node
e1
3, see the figure to the right. The elastic modulus of the rods is E. Rod e1
θ 3
has a cross sectional area A and a length l. Rod e2 is composed of two
cylindrical tubes to facilitate adjustment of its length by use of a hydrau-
lic actuator, where its length L is given by the angle ϕ. The effective cross 2
sectional area of rod e2 is 3A . The relations θ = 2 ϕ and L = 2l cos ϕ ϕ
are valid here. Determine the displacement of node 3 and the normal l m
force acting in rod e2 (the force acting on the hydraulic actuator), when
the crane is loaded by a mass m for the case ϕ = 30° . The acceleration e2 g
of gravity is known and denoted g.
1
Hint: the normal force can be determined by use the reaction forces act-
ing on node 2.
2P
2.10 A structure of three truss elements is loaded by two point
forces (P and 2P), see the figure to the right. The elastic modu-
lus, the cross sectional area and the length of each truss are E, 2A, 2L

E, A, L
shown in the figure. Analyse the structure by use of a matrix
formulated method and determine the reaction forces at all the
nodes. P
E, A, L

.
2.11 The truss structure to the right contains two truss elements and 90o
one spring element, with spring constant k = 2EA ⁄ L . The structure
is loaded by a point force P according to the figure. The truss ele-
k 45o
ments are of length L, have cross sectional area A and elastic modulus 45o
E. Determine the displacements at the nodes where the elements are EA
L P
connected. Evaluate also all reaction forces.
EA, L

The examples below are taken from “Exempelsamling i Hållfasthetslära” , Ed. P.-L. Larsson & R.
Lundell, KTH, Stockholm, january 2001.

2.12 Determine the displacements and the reaction


forces at the nodes. 3
Node x/L y/L 2 3k
2k
1 – 3 ⁄ 2 –1 ⁄ 2 y
2 1⁄2 P
– 3⁄2 x
k 4
3 0 1 1
4 0 0

– 2.3 (12) –
FEM for Engineering Applications—Exercises with Solutions / August 2008 / J. Faleskog

2.13 The planar truss structure to the right con-


sists of four spring elements, each of length L and 1 2
with spring constant k. All springs are oriented y
with a 45o angle with respect to the horizontal
plane. Determine all displacements and reaction P x
forces at the nodes. 5 3

2.14 Determine the displacements and the reac-


tion forces at the nodes, and the normal forces in
the springs. 1
k
Node x/L y/L
1 -1 1 y 2
2 1 0 x k
P
3 0 0 3

2.15 Determine the displacements and the reaction forces


at the nodes. y

1 2
Node x/L y/L
k-
------
k
1 0 2
--k- 2 4
2 1 2 2
k- Q
3 1 1 stel 3 ------
4 0 0 2 x

2.16 Determine the displacements and the reaction


forces at the nodes, and the normal forces in the 4L, k
spring elements. The stiffness and length of each 3
2
spring is shown in the figure to the right.
3L, 2k
5L, 5k
y
stel
x
1 Q

– 2.4 (12) –
FEM for Engineering Applications—Exercises with Solutions / August 2008 / J. Faleskog

Solutions

2k –k –k 0 0 R1

2.1 The equation system: – k 3k –k – k D 2 = P . Solution: D 2 = -----


P- 3
–k –k 3k –k D 3 0 D3 8k 1
0 –k – k 2k 0 R4
The reaction forces are obtained from Eqs. (1) and (4) as: R1 = R4 = – P ⁄ 2 .
2.2
D6 D4 Boundary conditions & prescribed forces:
k D1 = D4 = D5 = D6 = 0 and F2 = −P, F3 = 0
D5 D3
Equation- 16 12 – 16 – 12 0 0 0 R1
2k
5k system 12 19 – 12 –9 0 – 10 D 2 –P
D2
--k- –16 – 12 21 12 –5 0 D3 = 0
D1 5 –12 R4
–9 12 9 0 0 0
0 0 –5 0 5 0 0 R5
0 –10 0 0 0 10 0 R6
D2 P Reaction forces: R1 = – 4P ⁄ 17
Solution: = --------- – 7 R4 = 3P ⁄ 17
D3 17k – 4
R 5 = 4P ⁄ 17 R 6 = 14P ⁄ 17

2.3
D4
Element stiffness matrix: k e = ki r –r
D2 D3 –r r

D1 e2 Element 1: x k 1 = k, r= 1 0
y 0 0
1 x
e1 D8 D6
Element 2: −45o k 3 = 2k, r= 1 ⁄ 2 –1 ⁄ 2
e3
D5 –1 ⁄ 2 1 ⁄ 2
D7
x 2
B.C.: 2
D1 = D 2 = 0 D 3 = D4 = 0 Element 3:
45o k 2 = 2k, r = 1⁄2 1⁄2
1⁄2 1⁄2
D5 = D 6 = 0 D8 = 0 1
x
Assembly:
1 0 –1 0
0 0 R1 Reactions-
0 0 0 0
1 –1 –1 1 R2 forces
0 0
K = k1 + k2 + k 3 = k –1 1 1 –1 R3 Equation (7) gives
1 1 –1 –1 R4
0 0 1 1 –1 –1 F =
3kD 7 = P
R5
–1 0 –1 1 – 1 –1 3 0
⇒D
R6 P
0 0 1 –1 – 1 –1 0 2 = ------
P 7 3k
R8

– 2.5 (12) –
FEM for Engineering Applications—Exercises with Solutions / August 2008 / J. Faleskog

2.4
D2 D4 Boundary conditions:
D1 D3 D1 = D2 = D3 = D4 = D5 = D6 = D7 = D10 = 0

F8 = −P, F9 = −P
e1 D10 e2
D6
Element stiffness matrices:
D9 D5 2 c = cos φ
Ke = k i a –a , a = c sc ,
e4 e3
–a a 2 s = sin φ
sc s
D8
1
D7 e1& e4: k 1 = k 4 = 2k, a = { φ = – 45 ° } = --- 1 –1
2 –1 1

1
a = { φ = 45° } = --- 1 1
e2: k 2 = 2k, e3: k 3 = 4k, a = { φ = 90° } = 0 0
2 1 1 0 1
Assembly of global stiffness matrix:
Eqs. (8) & (9):
1 –1 –1 1
0 0 0

–1 1 1 –1
D8 D8 P
0
1 1
0 0
–1 –1 k 5 1 = –P = – --------- 2
1 1 –1 –1 1 3 D9 –P D9 14k 4
0 0 0 0
K = k 0 0 0
0 4 0 –4 Normal force in element 4: N = k 4 δ
0 0 1 –1 –1 1
0 0 0 –4 –1 5 1 –1
where k 4 = 2k and δ = u 2 – u 1
– 1 1 –1 –1
0
–1 1 3 –1 D9
1 –1 –1 –1 1 –1 –1 3
u1 1 D 10 1 D
= Tde = ------- 1 – 1 0 0 = – ------- 9
u2 2 0 0 1 –1 D7 2 D8

⇒ N = 3---------
7
2
-P
D8

– 2.6 (12) –
FEM for Engineering Applications—Exercises with Solutions / August 2008 / J. Faleskog

2.5
Boundary conditions: D1 = D2 = D3 = D4 = D5 = D7 = 0; F6 = 0; F8 = −P
D2
D4 2 c = cos φ
e1 D1
e2
Element stiffness matrices: K e = k i a –a , a = c sc ,
D3 –a a 2 s = sin φ
sc s
EA 1
D6 e1: k 1 = -------, a = { φ = – 45 ° } = --- 1 –1
L 2 –1 1
D5
e3
2EA
e2: k 2 = -----------, a = { φ = 90° } = 0 0
L 0 1
D8
EA 1
D7 a = { φ = 45° } = --- 1 1
e3: k 3 = -------,
L 2 1 1
Assembly of stiffness:
Eqs. (6) & (8) (reduced system of equations):
1 –1 0 –1 1 0
–1 1
0 0 0
1
0
–1
0
0
EA
------- 6 –1
2L – 1 1 D
D6
= 0
–P
⇒ D6
D8
PL
= – ----------- 2
5EA 12
EA 0 4 0 –4 8
K = -------
2L – 1 1 0 0 2 0 –1 –1
Reaction forces in node 1 & 2 ( D.O.F.:s 1 - 4):
1 –1 0 –4 0 6 –1 –1
EA P EA P
–1 R 1 = ------- D 6 = – ---
–1 1 1 R 2 = ------- ( – D 6 ) = ---
0 0 –1 –1
2L 1 1
5 2L 5
R3 = 0 4EA 4P
R 4 = ----------- ( –D 6 ) = -------
2L 5
P 4P
Global equilibrium in vertical dir.: R 2 + R4 + F6 + F 8 = + --
- ------
- + 0 – P = 0 OK!
5 5
2.6
2 Boundary conditions: D1x=D1y=D2x=D2y=D3x=D3y=D4x=D4y=0
2 The element stiffness contribution to node 5:
e2 e1: EA
------- 1 0 ; e2: EA
------- 0 0 ;
2 3 L 0 0 L 0 1
1 e3
1 φ
1
e1
2
e3: η EA
2 c = cos φ
5 1 ------- c sc
1
π
L 2 s = sin φ
⎫⎪
sc s
–φ
⎛⎝ ⎛⎝ ⎞⎠ ⎞⎠
e4 ---
2 π
2
cos – --- – φ = sin φ
2 ⎪⎬ ⇒ η EA 2
------- s – s c
⎛⎝ ⎛⎝ ⎞⎠ ⎞⎠ ⎪⎪ L –sc c
4 e4:
π 2
sin – --- – φ = – cos φ
2 2
2

K = ------- 1 + η ( c + s ) η ( sc – sc )
Assembly: EA EA
= ( 1 + η ) ------- 1 0
(only node 5) L 2 2 L 0 1
η ( sc – sc ) 1 + η ( c + s )

Eq. system:
EA
( 1 + η ) ------- 1 0
L 0 1 D
D 5x
= P cos α
sin α
⇒ D 5x
D 5y
= ------------------------- cos α
PL
( 1 + η )EA sin α
5y
D5y δ
Thus: α
D5x
⇒ δ = -------------------------
PL
( 1 + η )EA

– 2.7 (12) –
FEM for Engineering Applications—Exercises with Solutions / August 2008 / J. Faleskog

2.7
D4 D6 Boundary Condistions:
D3 D5 D1 = D2 = D3 = D4 = D5 = D6 = 0; F7 = Pcosϕ; F8 = Psinϕ

e4 D10 e3 P Element sttiffness matrix:


D2 ϕ 2 c = cos φ
e1 K e = k i a –a , a = c sc ,
–a a 2 s = sin φ
D1 D9 e2 sc s

3 1
e1: k 1 = --- k 0, a = { φ = 0° } = 1 0 e3: k 3 = k 0, a = { φ = 60° } = --- 1 3
4 0 0 4
3 3
1
e2: k 2, a = { φ = 0° } = 1 0 e4: k 2 = k 0, a = { φ = 120° } = --- 1 – 3
0 0 4
– 3 3
EA
where k 0 = -------
L
Assembly Ki and implementation of B.C. gives the reduced Equation system:

1.25k 0 + k 2
0
0
1.5k 0 D 10
D9
= P cos ϕ
sin ϕ
But D should aligned
with the external force
⇒ D 10
D9
= D 0 cos ϕ
sin ϕ

Eq. 9: 1.25k 0 + k 2 = P ⁄ D 0 ⎫⎬ ⇒ ⇒k EA

1.25k 0 + k 2 = 1.5k 0 2 = 0.25k 0 = -------
Eq. 10: 1.5k 0 = P ⁄ D 0 4L

2.8
D2
Boundary Conditions: D1 = D2 = D3 = D4 = D5 = D6 = 0
D1
y F7 = – mg sin ϕ, F 8 = – mg cos ϕ
a i – ai
e1 Element stiffness matrix: Ki = k i
–a i a i
D8
D7 where
x 1 1
e2 e3 a 1 = 0 0 , a 2 = --- 3 3 , a 3 = --- 3 – 3
D4 0 1 4 4
D6 3 1 – 3 1
D3
ϕ
D5
mg
Assembly of the stiffness matrix:
a1 0 0 – a1 Eq. (7) & (9):


0 a2 0 – a2
= – mg sin ϕ = – ----------- sin ϕ
K = k 3--- 1 0 D 7 D7 2mg
k
0 0 a3 –a 3 2 0 1 D cos ϕ D8 3k cos ϕ
8
–a1 –a2 –a3 a1 + a2 + a3

– 2.8 (12) –
FEM for Engineering Applications—Exercises with Solutions / August 2008 / J. Faleskog

2.9
D6
Boundary Cond.: D1 = D2 = D3 = D4 = 0, F 5 = 0, F6 = – mg
o D5
60 e1
D4 Element stiffness matrices:
D3 a i – ai EA EA
mg Ki = where a 1 = ------- 3 3 , a 2 = ------- 1 3
–ai ai 4l 4l
30o e2 3 1 3 3
y Assembly of stiffness matrix: 1 3 0 0 –1 – 3
D2
x 3 3 0 0 – 3 –3
D1 a2 0 – a2
EA 0 0 3 3 –3 – 3
K = 0 a1 –a1 = -------
4l
0 0 3 1 – 3 –1
–a2 –a1 a1 + a2
–1 – 3 –3 – 3 4 2 3
– 3 –3 – 3 –1 2 3 4

Eq. (5) & (6):


EA- 2 3 D 5
------
2l
= –mg 0 ⇒ D5 2mgl
= ------------- 3
EA – 2
3 2 D6 1 D6
Reaction forces EA 3 EA 3
at node 1: R1 = ------- ( – 1D 5 – 3D 6 ) = ------- mg, R 2 = ------- ( – 3D5 – 3D 6 ) = --- mg
4l 2 4l 2
R2

N2
Equilibrium in y-dir. gives: R2 + N 2 cos 30 = 0 N 2 = – 3mg
R1
2.10
D4 B.C.: D1 = D2 = D3 = D6 = 0, F4 = 2P, F5 = −P
2
D3
2 2 c 2 = cos φ
Element stiffness matrices: K e = k i a – a , a = c sc ,
e2 –a a 2 s = sin φ
e3 sc s
EA 1 0
1
D2 1 D6 e1: k 1 = -------, a = { φ = 0° } =
D1 e1 D
L 0 0
5
1 1 2 3
EA EA 1
e2: k 2 = -------, a = { φ = 90° } = 0 0 e3: k 2 = -------, a = { φ = 135° } = --- 1 –1
L 0 1 L 2 –1 1

2 0 0 0 –2 0 Eqs. (4) & (5):


0 2 0 –2 0 0
Assembly of
stiffness:
EA
K = ------- 0
2L 0
0
–2
1
–1
–1
3
–1
1
1
–1
EA 3 1 D 4
-------
2L 1 3 D
= 2P
–p
⇒ D4
D5
PL
= ----------- 7
4EA – 5
5
–2 0 –1 1 3 –1
0 0 1 –1 –1 1
Reaction forces:
(1):
EA 5P
R 1 = ------- ( – 2D 5 ) = ------- ⎫⎪ (3):
EA
2L
P
R 3 = ------- ( – D 4 – D 5 ) = – ---
4
(Node 2)
2L 4

= ------- ( – 2D ) = ---------- ⎪
(Node 1)

EA – 7P EA P
(2): R2 4 (6): R 6 = ------- ( – D 4 – D 5 ) = – --- (Node 3)
2L 4 2L 4

– 2.9 (12) –
FEM for Engineering Applications—Exercises with Solutions / August 2008 / J. Faleskog

2.11
D6 D4
2 Boundary conditions: D2 = D3 = D5 = D6 = 0,
D5 e3
3 D3
1 2 F1 = P, F4 = 0
1 2
e1 e2
D2
Element stiffness 2 c = cos φ
1 K e = k i a –a , a = c sc ,
2 D1 matrices: –a a 2 s = sin φ
sc s
1

EA 1 1 –1 EA 1 1 1
e1: k 1 = -------, a = { φ = – 45 ° } = --- e2: k 2 = -------, a = { φ = 45° } = ---
L 2 –1 1 L 2 1 1
2EA
e3: k 3 = -----------, a = { φ = 0° } = 1 0
L 0 0
2 0 –1 –1 –1 1
Eqs. (1) & (4):
0 2 –1 –1 1 –1
EA – 1

Assembly: –1 5 1 –4 0
K = ------- EA 2 – 1 D 1 D1 2PL
2L – 1 –1 1 1 0 0 ------- = P = ---------- 1
2L – 1 1 D 0 D4 EA 1
4
–1 1 –4 0 5 –1
1 –1 0 0 –1 1

EA EA
Reaction forces: (2): R 2 = ------- ( – D 4 ) = – P (Node 1) (5): R 5 = ------- ( – D 1 ) = – P (Node 3)
2L 2L
EA EA
(3): R 3 = ------- ( – D 1 + D 4 ) = 0 (Node 2) (6): R 6 = ------- ( D 1 ) = P (Node 3)
2L 2L

2.12
6 Boundary conditions:
3 3 0 0 0 0 –3 – 3
4 5 3 1 0 0 0 0 – 3 –1 D 1 = D2 = 0
0 0 6 –2 3 0 0 –6 2 3 D 3 = D4 = 0
3 k
8 K = --- 0 0 –2 3 2 0 0 2 3 –2
D 5 = D6 = 0
4
0 0 0 0 0 0 0 0
7 0 0 0 0 0 12 0 – 12 F 7 = P, F 8 = 0
2
–3 – 3 –6 2 3 0 0 9 – 3

1 – 3 –1 2 3 –2 0 – 12 – 3 15

F1 – 12 – 0.3636
F2 –4 3 – 0.2099
F3 P – 21
Equation- Reaction- = P – 0.3664
⇒ D7 = ------
P 15 P
= --- 0.4545
–1 33 7 3
D = K F = --------- F4 0.3674
system: D8 33k 3 k 0.0525 forces:
F5 0 0
–3 3 – 0.1575
F6

– 2.10 (12) –
FEM for Engineering Applications—Exercises with Solutions / August 2008 / J. Faleskog

2.13

1 –1 0 0 0 0 0 0 –1 1 Boundary conditions:
–1 1 0 0 0 0 0 0 1 –1 D1 = D2 = 0
2 4 0 0 1 1 0 0 0 0 –1 –1
0 0 1 1 0 0 0 0 –1 –1 D3 = D4 = 0
1 3 k 0
K = --- 0 0 0 1 1 –1 –1 0 0 D5 = D6 = 0
2 0 0 0 0 1 1 –1 –1 0 0
10 6
0 0 0 0 –1 –1 2 0 –1 1 D8 = 0

9 5 0 0 0 0 –1 –1 0 2 1 –1 F 7 = 0, F 9 = – P, F 10 = 0
–1 1 –1 –1 0 0 –1 1 3 –1
8
1 –1 –1 –1 0 0 1 –1 –1 3

7
F1
2
F2
–2
D7
Equation- Reaction- F3

–2 3
–1 P P
D = K F D9 = ------ – 5 F4 = --- 3
system: 6k forces: 6
D 10 –1 F5 1
F6 1
–2
F8

2.14
2 Boundary conditions:
1 –1 0 0 –1 1
1 –1 1 0 0 1 –1 D1 = D2 = 0
1 k
K = --- 0 0 2 0 –2 0
D3 = D4 = 0
e1 2 0 0 0 0 0 0
6 4
–1 1 –2 0 3 –1 D6 = 0
2 5 e2 3 1 –1 0 0 –1 1
F 5 = –P
1 2
Equation-
system:
–1
D = K F ⇒ 2P
D 5 = ------- – 1
3k
Reaction- F1
F2
1
forces: P
–1
F3 = --- 2
3
F4 0
F6 1
The normal force, N, in one element is given by f e = kTD e ,
where N = f2 .

Element 1: D1

–1 1
1
k = k 1 –1 ; T = ------- 1 – 1 0 0 ; D e =
2 0 0 1 –1
D2
D5
⇒ f = ------k2- –DD –+DD –+DD +–DD ⇒ N = – ------32-P
1 2 5 6

1 2 5 6
D6

Element 2: D5

k = k 1 –1 ; T = 1 0 0 0 ; D e =
–1 1 00 1 0
D6
D3
⇒ f = k –DD –+DD ⇒ N = 2--3- P
5 3

5 3
D4

– 2.11 (12) –
FEM for Engineering Applications—Exercises with Solutions / August 2008 / J. Faleskog

2.15
4
2
1 –1 0 0 0 0 –1 1
1 3 –1 1 + 2 0 0 0 – 2 1 –1
8 0 0 0 0 0 0 0 0
k
7 K = ---------- 0 0 0 2 2 0 0 0 –2 2
2 2 0 0 0 0 1 1 –1 –1
6 0 – 2 0 0 1 1 + 2 –1 –1
stel –1 1 0 0 –1 –1 2 0
5
1 –1 0 –2 2 –1 –1 0 2+2 2

D6 –6+4 2
Boundary conditions:

Equation- – 0.3431
–1 Q Q
D = K F D7 = ---- – 3 + 2 2 = ---- – 0.1716
D1 = D2 = 0 system: k k
D8 5–4 2 – 0.6569
D3 = D4 = 0
F1 –3 2+4
D5 = 0 (p.g.a. stel bom) – 0.1716
Reaction- F2 6 2–8 0.3431
F 6 = F 7 = 0, F 8 = – Q forces Q
F3 = ------- 0 = Q 0
2
F4 8–5 2 0.6569
F5 0.1716
3 2–4

2.16
6 4
16 12 – 16 – 12 0 0 Boundary conditions:
5 12 19 – 12 –9 0 – 10 (rigid support)
3 D1 = 0
2 1 e2 2 k – 16 – 12 21 12 –5 0
K = --- D4 = 0
e3 2 5 – 12 –9 12 9 0 0
1 2 e1 0 0 –5 0 5 0 D5 = D 6 = 0
“rigid” 1 0 – 10 0 0 0 10 F 2 = – Q, F 3 = 0
1
F1
Equation- Reaction- – 20 – 0.2353
system: D –1
= K F ⇒ D2
D3
Q
17k – 4
Q
= --------- – 7 = ---- – 0.4118
k – 0.2353
forces F4 Q 15
= ------
85 20
= Q 0.1765
F5 0.2353
F6 70 0.8235

The normal force, N, in one element is given by f e = kTD e ,


where N = f2 .
D1

Element 1:
–1 1
1
k = 5k 1 – 1 ; T = --- 4 3 0 0 ; D e =
5 0 0 43
D2
D3
⇒ f = k –3D – 4D
3D + 4D
⇒ 5 2
N = ------ Q
17
3

2 3
D4

D5

Element 2:
–1 1
1
k = k 1 – 1 ; T = --- 1 0 0 0 ; D e =
5 0 0 1 0
D6
D3
⇒ f = k –DD ⇒ N = – 3 4
------ Q
17
3
D4

D1

Element 3: k = 2k 1 – 1 ; T = 0 1 0 0 ; D e =
–1 1 0 00 1
D2
D5
⇒ f = 2k –DD ⇒ N = 14-----17- Q
2

2
D6

– 2.12 (12) –
FEM for Engineering Applications—Exercises with Solutions / August 2008 / J. Faleskog

3. Strong/weak form and FEM-equations

3.1 The solution to a specific one-dimensional problem is governed by the differential equa-
tion (strong form)

d- ⎛ dφ
----- D ------⎞ – qφ + Q = 0 for x 1 ≤ x ≤ x 2 ,
dx ⎝ dx ⎠

where the primary variable φ depends on x. Also D, q and Q may depend on x. Derive the
weak form and identify the essential and natural boundary conditions.

d dφ
3.2 The weak form to ------ ⎛⎝ D ------⎞⎠ – qφ + Q = 0 is
dx dx

x2 x2 x2
x2
dv
------ D dφ dφ
∫ dx dx
------ dx + ∫ vqφdx = ∫
vQdx + vD ------ ,
dx x1
x1 x1 x1

where v(x) is an arbitrary weight function. Derive the FEM-equation for one element. Use a
linear interpolation for the primary variable and use Galerkin’s method, regarding the choice
of the weight function.

3.3 The figure to the right shows a rod with elastic


modulus E and cross sectional area A. The rod is E, A Kx
x
loaded by a body force, Kx [N/m3]. The displace-
x=0 x=L
ment, u, in the rod is given by the solution to the dif-
ferential equation
d- ⎛ du
----- EA ------⎞ + AK x = 0 .
dx ⎝ dx⎠
L L
dv du L
(a) Show that the weak form is ∫ ------ EA ------ dx = ∫
vK x Adx + [ v ( σA ) ]0 ,
dx dx
0 0
where σ denotes the normal stress and v is an arbitrary weight function.
(b) Derive the FEM-equation (use Galerkin’s method) to the weak form above for one ele-
ment, i.e. identify the quantities in the equation
k e d e = fe . Kx
(c) The rod shown to the right is of length 3L and E, A x
loaded by K x = Q ⁄ ( 2AL ) ( x ⁄ L – 1 ) , where
Q corresponds to the total axial force acting x=0 x=L x = 3L
on the rod. Both ends of the rod are clamped. ⎧ 2 QL x
Exact ⎪ --- -------- --- 0≤x≤L
Divide the rod into two elements of lengths L ⎪ 9 EA L
and 2L respectively and determine the node soln. u ( x ) = ⎨
-------- 3 – --x-
1- QL x 2 x 3
⎪ ----- + 9 ⎛ ---⎞ – 3 ⎛ ---⎞
⎪ 36 EA ⎝ L⎠ ⎝ L⎠
displacements and the reaction forces. Com- ⎩
L
pare with the exact solution. Redo the analy- 2Q 7Q
N ( x = 0 ) = ------- , N ( x = 3L ) = – -------
sis with more elements! 9 9

– 3.1 (15) –
FEM for Engineering Applications—Exercises with Solutions / August 2008 / J. Faleskog

3.4 The right figure shows a uniaxial bar coupled to


a set of continues springs with spring constant per
unit length kx [(N/m) / m]. The bar has elastic modu- E, A Kx
lus E, cross sectional area A and is loaded by a body x
force Kx [N/m3]. The displacement, u, in the bar is
given by the solution to the differential equation x=0 kx x=L

------ ⎛ EA du
d
------⎞ – k x u + AK x = 0 .
dx ⎝ dx⎠
(a) Show that the weak form is
L L L
dv
------ EA du
L
∫ dx
------ dx + ∫ vk x udx = ∫ vK x Adx + [ v ( σA ) ] 0 ,
dx
0 0 0
where σ denotes the normal stress and v is an arbitrary weight function.
(b) Derive the FEM-equation (use Galerkin’s method) to the weak form above for one ele-
ment, i.e. identify the quantities in the equation

ke d e = f e .
(c) Divide the bar into two linear elements of the same length and analyse the problem.
Evaluate the node displacements. Apply the boundary conditions: u = 0 for x = 0 and
σA = Q for x = L. Assume that E and A are constants and that the spring constant
2
k x = 3EA ⁄ ( 2L ) and the body force K x = Q ⁄ ( AL ) .

3.5 Figure (a) to the right shows a uniform bar loaded (a) (b)
by its dead weight, ρg, where ρ is the density of the bar
and g is the acceleration of gravity. The bar has elastic
x EA
modulus E and cross sectional area A. The displace- x
ρ
ment, u, is given by the solution to the differential x=L
equation g
P
d du x = 2L
------ ⎛ EA ------⎞ + A ρ g = 0 .
dx ⎝ dx⎠ k
x=L
x = 3L
(a) Assume that E, A, ρ and g are constants and
show that the weak form is
L L
dv du L
EA ∫ ------ ------ dx = [ v ( σA ) ]0 + ρ gA ∫ vdx ,
dx dx
0 0

where σ is the normal stress in the bar and v an arbitrary weight function.
(b) Derive the FEM-equation (use Galerkin’s method) to the weak form above for one ele-
ment, i.e. identify the quantities in the equation

ke d e = f e .

– 3.2 (15) –
FEM for Engineering Applications—Exercises with Solutions / August 2008 / J. Faleskog

(c) In an application, a bar (E, A) is connected to a linear spring with spring constant k, see
Figure (b) above. The bar is loaded by a point force applied at the x = L and by its dead
weight. Divide the bar/spring structure in three elements with nodes placed in the
points: x = 0, L, 2L and 3L. Thus, the bar should be divided into two equal elements.
Let k = 2EA ⁄ L , where EA is constant, and calculate the displacements at the nodes.

3.6 The figure to the right shows a beam with bending


stiffness EI attached to an elastic foundation character-
z,w
ized by a spring constant per unit length kz [(N/m) / m]. A
distributed load per unit length q [N/m] is applied on the q
beam. The deflection of the beam w is given by the solu-
x
tion to the differential equation

2 2 x = −L kz x=L
d - ⎛ ---------
d w⎞
------- ⎜ EI ⎟ + kz w – q = 0 .
2 2
dx ⎝ dx ⎠
T
M T M

(a) Show that the weak form is


L L L
2 2 L L
d v- d---------
w dv

-------
2
EI 2
dx + ∫
vk z wdx = [ vT ] – ------ M + ∫
vqdx ,
dx dx –L dx
–L –L –L –L
where v is an arbitrary weight function, T is a shear force and M is a moment. The rela-
tions T = – ( EIw″ )′ and M = – EIw″ have been utilized at the boundaries.

(b) Derive the FEM-equation (use Galerkin’s method) to the weak form above for one ele-
ment, i.e. identify the quantities in the equation ke d e = f e .

z,w
(c) Divide the beam into a two-node beam element (2
x M0
D.O.F. per node) and determine w′ ( L ) . Assume
4
that EI is constant, k z = λEI ⁄ L and q = 0. The
boundary conditions are shown in the figure to the x = −L x=L
right.

(d) The beam shown below is subjected to a uniformly distributed load q = Q ⁄ ( 2L ) ,


where Q is the resultant of the total distributed load acting on the beam. The total
length of the beam is 2L and its bending stiffness is EI. The left end of the beam is
clamped, whereas the right end support is flexible, here modelled by a combination of a
tension spring with stiffness kw [N/m] and a torsion spring with stiffness kθ [Nm].
Model the beam with one beam element (2 node element with 2 D.O.F. per node) and
calculate the deflection and rotation of the right end of the beam. Use the values of the
spring constants shown in the figure.
Q kθ 3EI EI
k w = --------3- k θ = ------
2L L
2L, EI
kw

– 3.3 (15) –
FEM for Engineering Applications—Exercises with Solutions / August 2008 / J. Faleskog

3.7 The figure to the right shows a beam, which is loaded


z,w
by its dead weight per unit volume ρg, where ρ is the den- g
sity and g acceleration of gravity. The beam has the elas- x
tic modulus E, moment of inertia I and cross sectional
area A. The deflection of the beam w is given by the solu- x = −L x=L
tion to the differential equation T

2 2 M T M
d - ⎛ ---------
d w⎞
------- ⎜ EI ⎟ + ρ gA = 0 .
2 2
dx ⎝ dx ⎠
(a) Assume that ρ, g and A are constants and show that the weak form is
L L
2 2 L L
d-------v- d---------
w dv
∫ 2
EI 2
dx = [ vT ] – ------ M – ρ gA ∫ vdx ,
dx dx –L dx
–L –L –L
where v is an arbitrary weight function, T is shear force and M is moment (defined
according to the figure above). The relations T = – ( EIw″ )′ and M = – EIw″ have
been utilized at the boundaries.
(b) Derive the FEM-equation (use Galerkin’s method) to the weak form above for one ele-
ment, i.e. identify the quantities in the equation ke d e = f e .
(c) The figure to the right shows a cantilever beam
attached to a linear spring with spring constant g z,w
3
k = η EI ⁄ L . The beam is loaded by its dead weight k
and by a point force P. Analyse the beam by use of a x
two-node beam element and calculate its deflection
P
and rotation (slope) at x = L for the special case that x = −L
η = 1 ⁄ 2 , P = ρ gAL and EI = constant. x=L

3.8 The figure to the right shows a beam with bending stiff-
z,w Q
ness EI subjected to a distributed triangular load acting down-
wards with a total resultant force equal to Q. The beam has x
elasticity modulus E and moment of inertia I. The deflection
x = −L x=L
of the beam (vertical displacement), w, is given by the solu- T
tion to the differential equation
M T M
2 2
d - ⎛ ---------
d w⎞ ⎛ x⎞ Q
------- ⎜ EI ⎟ + 1 + --- ------ = 0
2 2 ⎝ L⎠ 2L
dx ⎝ dx ⎠
(a) Show that the weak form to the differential equation is
L L
2 2 L
d-------v- d---------
w L dv 1 x dx

EI dx = [ vT ] – ------ M –Q∫ v --- ⎛ 1 + ---⎞ ------ ,
2 2 –L dx 2 ⎝ L⎠ L
dx dx –L
–L –L
where v is an arbitrary weight function, T is a shear force and M is a moment (see the
figure above). The relations T = – ( EIw″ )′ and M = – EIw″ have been used at the
boundaries.

(b) Derive the FEM equation of the weak form above for one element, i.e. identify the
quantities in the equation ke d e = f e (use Galerkin’s method).

– 3.4 (15) –
FEM for Engineering Applications—Exercises with Solutions / August 2008 / J. Faleskog

(c) In an application, the beam is clamped at x = L and z,w x=L


the rotation is prevented at x = −L, see the figure x = −L
Q x
to the right. The beam is subjected to the triangu-
lar load and a point force according to the figure. 2L, EI
P
Analyse the beam by use of one 2-node beam ele-
ment and calculate the deflection w(ξ), where ξ is natural coordinate defined as
ξ = x ⁄L.
3.9 A one dimensional model of a cooling fin is shown
to the right. The cooling fin has a cross sectional area A Convection
[m2], length 3L/2 and coefficient of thermal conductiv- Heat conduction
x
ity k [W/m/oC]. The convection coefficient is h [W/m2/
o
C] and the perimeter of the fin is P [m]. The tempera- T(x)
x = 0 x = 3L ⁄ 2
ture distribution in the fin T [oC] at steady state condi-
tions is given by the solution to the differential equation

d- ⎛ dT
----- kA ------⎞ + qA – hP ( T – T∞ ) = 0 .
dx ⎝ dx ⎠
Here, q [W/m3] is a continues distributed heat source and T∞ is the ambient temperature (the
last term represents convection to the surrounding medium).
(a) Show that the weak form to the differential equation is
3L ⁄ 2 3L ⁄ 2 3L ⁄ 2
dv 3L ⁄ 2

------ kA dT
------ dx + ∫
vhPTdx = [ v ( – Q ) ] + ∫
v ( qA + hPT ∞ )dx ,
dx dx 0
0 0 0
where v is an arbitrary weight function and Q is heat flow, where Q = – k AdT ⁄ dx
(Fourier’s law) has been used at the boundaries.
(b) Derive the FEM-equation (use Galerkin’s method) to the weak form above for one ele-
ment, i.e. identify the quantities in the equation ke d e = f e .
(c) Divide the cooling fin into three linear elements (two nodes per element and one tem-
perature d.o.f. per nod) and determine the temperature at the nodes. The boundary con-
ditions are described by T = 4T ∞ at x = 0 and T = T∞ at x = 3L ⁄ 2 . Assume that k,
2
A and P are constants, q = 0 and that hP = 12kA ⁄ L .
(d) Change the boundary conditions in x = 3L ⁄ 2 from prescribed temperature to convec-
tion. Assume that the relation between the surface of the perimeter and the surface of
the end of the fin is LP = A ⁄ 96 .

3.10 The figure to the right shows a model for heat con-
duction in a one-dimensional rod, where heat exchange Convection
by convection between the surface of the rod and the Heat conduction
surrounding medium is taken into consideration. The x
ambient temperature of the surrounding medium is T ∞ . x = x
1 T(x) x = x2
The rod has cross sectional area A [m2], perimeter P
[m], thermal conductivity k [W/m/oC] and convection
heat transfer coefficient h [W/m2/oC]. The temperature T [oC] in the rod as a function of posi-
tion at steady state conditions is given by the solution to the differential equation

– 3.5 (15) –
FEM for Engineering Applications—Exercises with Solutions / August 2008 / J. Faleskog

d- ⎛ dT
----- kA ------⎞ – hP ( T – T ∞ ) = 0 .
dx ⎝ dx ⎠

(a) Show that the weak form to the differential equation is


x2 x2 x2
dv dT x
------ kA ------ dx + ∫ vhPTdx = [ v ( – Q ) ] x 2 + ∫ vhPT∞ dx ,
∫ dx dx 1

x1 x1 x1

where v is an arbitrary function of x and Q is heat flow, where Q = – k AdT ⁄ dx (Fou-


riers law) has been used at the left and right boundary of the rod.
(b) Derive the FEM equation of the weak form above for one element, i.e. identify the
quantities in the equation ke T e = fe (use Galerkin’s method).
(c) Assume that the total length of the rod is L and that k, h, A and P are constants, related
as hPL = 16kA ⁄ L . Divide the rod into two equal linear elements (two nodes per ele-
ment with one temperature d.o.f. per node) and calculate the temperature at the nodes.
The boundary conditions are described by a prescribed heat flow Q = hPLT ∞ at
x = 0 and a prescribed temperature T = T∞ at x = L .

FORMULAS

1D: φ1
φ ( ξ ) = N1 φ 1 + N 2φ 2 = N1 N2 N1 = 1 – ξ N2 = ξ
1 φ2
L 2
φ1 φ2 N
ξ L L
T
L dN N- --1- 1 – 1
0 1
T
--- 2 1
∫ N Ndx = { dx = Ldξ } = 6
---------- -----
∫ dx dx dx = L
1 2 –1 1
0 0

2 2
Beam element: d N 1d N
Deflection: w ( ξ ) = N 1 d 1 + N 2 d 2 + N 3 d 3 + N 4 d 4 = Nd e , B = ---------2- = -----2 ---------2-
dx L dξ
d1 d3 3 2 3
2L, EI N 1 = ( 2 – 3ξ + ξ ) ⁄ 4, N2 = L ( 1 – ξ – ξ + ξ ) ⁄ 4
3 2 3
d2 d4 N 3 = ( 2 + 3ξ – ξ ) ⁄ 4, N4 = L ( – 1 – ξ + ξ + ξ ) ⁄ 4
ξ
−1 0 1
3 3L – 3 3L 78 22L 27 – 13L
L L
2 2
T 1 - 3L 4L – 3L 2L
-------- T L - 22L 8L 13L – 6L 2
--------
2
∫ B Bdx = 3 ∫ N Ndx =
105 27 13L 78 – 22L
2L – 3 – 3L 3 – 3L
–L –L
2 2 2 2
3L 2L – 3L 4L – 13L – 6L – 22L 8L

L 9
1--- ⎛
T x---⎞ dx
------ 1
------ 4L
∫ N 2 ⎝ 1 + L⎠ L = 30
21
–L
– 6L

– 3.6 (15) –
FEM for Engineering Applications—Exercises with Solutions / August 2008 / J. Faleskog

Solutions

x2
d dφ
3.1 (i) Weighted residual: v ⎛ ------ ⎛ D ------⎞
∫ ⎝ dx ⎝ dx ⎠
– qφ + Q⎞ dx = 0 , where v is an arb. weight fcn.

x1
x2 x2
x2
d dφ dφ dv dφ
(ii) Integration by parts ∫ v ⎛ ------ ⎛ D ------⎞ ⎞ dx = vD ------ – ∫ ------ D ------ dx gives
⎝ dx ⎝ dx ⎠ ⎠ dx x1 dx dx
x1 x1

Weak form:
x2 x2 x2
x2
dv
------ D dφ dφ dφ dφ
∫ dx dx
------ dx + ∫ vqφdx = ∫
vQdx + vD ------ φ = φ 0 eller D ------ = D ------ på x = x i
dx x1 dx dx 0
x1 x1 x1
Essential B.C.
Natural B.C.

φ1
3.2 Approximation function: φ(ξ) = Nφ e , N = 1 – ξ ξ , φe =
φ2

⇒ ------ = dφ
------ dξ
------ = 1--- dN
------- φ = Bφ e , where l is the element length
dx dξ dx l dξ e
T T T
Weight fcn. (Galerkin’s method): v ( ξ ) = Nβ = β N where β = β 1 β 2 (arbitrary)
T
dv dv- -----
dξ- T 1 dN T T
------ = -----⇒ = β --- ---------- = β B
dx dξ dx l dξ
Inserted into the weak form gives
1 1 1
T T T T T T dφ 1
β ∫
B DBldξ + ∫ N qNldξ φ e = β ∫
N Qldξ + N D ------
dx 0
0 0 0

kD kq fQ fs

βT is an arbitrary vector ⇒ k e φ e = f e där k e = kD + kq och f e = fQ + f s


The element matrices becomes:
1 1 1
1 1 1
= ∫ B DBldξ = ∫ --- –1 D --- – 1 1 ldξ = --- 1 – 1 ∫ Ddξ
T
kD
l 1 l l –1 1
0 0 0
1 1 1
2
T 1–ξ q ( 1 – ξ ) ( 1 – ξ )ξ qdξ
kq = ∫
N qNldξ = ∫ 1 – ξ ξ ldξ = l ∫
ξ 2
0 0 0 ( 1 – ξ )ξ ξ
1 1

N Qldξ = l ∫ 1 – ξ Qdξ
T
fQ = ∫
ξ
0 0

3.3 (a) See solution to 3.1 and 3.2

– 3.7 (15) –
FEM for Engineering Applications—Exercises with Solutions / August 2008 / J. Faleskog

L T L T L
T
3.3 (b) k e de = f e , where k e = ∫
B EAB dx and fe = ∫
N K x A dx + [ N ( σA ) ] .
0
0 0
3.3 (c) Node/element division: D1 D2 D3 2 QL
D 2 = --- --------
9 EA
2 –2 0 0 R1 2
Eqn. system: EA ⇒ R 1 = – --- Q
------- – 2 3 –1 D 2 = Q⁄3 9
2L
0 –1 1 0 R 3 + 2Q ⁄ 3 7
R 3 = – --- Q
9
3.4(a)
L
d du
Weighted residual: ∫
v ------ ⎛⎝ EA ------⎞⎠ – k x u + AKx dx = 0 (1)
dx dx
0
L L
d du du L dv du
Integration by parts: ∫ v ------ ⎛ EA ------⎞ dx = v ⎛ EA ------⎞ – ∫ ------ EA ------ dx (2)
dx ⎝ dx⎠ ⎝ dx⎠ 0 dx dx
0 L L L 0
(2) inserted into (1) L
dv
------ EA du
du
gives the weak form: ∫ dx
------ dx + ∫ vk x udx = ∫
vAK x dx + v ⎛⎝ EA ------⎞⎠
dx dx 0
0 0 0

du
3.4(b) Displacement interpolation: u = Nd e ------ = dN------- de = Bd e
dx dx
T
T T dv T dN T T
Weight function: v = Nb e = b e N -----
- = be ---------
- = be B
dx dx

T T T T T T Le
be ∫
B EAB dx + ∫
N k x N dx de = b e ∫
N K x A dx + [ N ( σA ) ] 0
Le L Le
⎧ ⎪ ⎪ ⎪ ⎪ ⎨ e⎪ ⎪ ⎪ ⎪ ⎩ ⎧⎪ ⎪ ⎪ ⎪ ⎨ ⎪ ⎪ ⎪ ⎪ ⎩
ke fe
T
but b e is arbitrary ⇒ k e d e = fe

3.4(c)
Le = L/2 Le = L/2 2
N = (1 – ξ) ξ B = --- – 1 1
D1 D2 D3 L
1

L 2EA
B EAB --- dξ = ----------- 1 –1
T ⎪
Element- ∫ 2 L –1 1 ⎪ 1
matrices: ⎪ EA T Q L Q 1
ke = ------- 18 – 15 ; ∫ N ------- A --- dξ = ----
0
⎬⇒ AL 2 4 1
1

8L –15 18
T 3EA L
- N --- dξ = EA
0

N ---------- ------- 2 1 ⎪
2 2 8L 1 2 ⎪
2L ⎭
0 reaction force
18 –15 0 D 1 1 R
EA
Assembly: ------- – 15 36 –15 D 2 = Q
---- 2 + 0
8L 4 point force
0 –15 18 D 3 1 Q

D2 QL QL
Equation (2) & (3): ⇒ = ----------------- 74 = -------- 0.525
D3 141EA 140 EA 0.993

– 3.8 (15) –
FEM for Engineering Applications—Exercises with Solutions / August 2008 / J. Faleskog

3.5(a)
L
d du
Weighted residual: ∫ v ------ ⎛⎝ EA ------⎞⎠ + Aρg dx = 0 (1)
dx dx
0
L L L
d du du dv du
Integration by parts: v ------ ⎛ EA ------⎞⎠ dx = v ⎛ EA ------⎞
∫ dx ⎝
– ∫ ------ EA ------ dx (2)
dx ⎝ dx⎠ dx dx
0
0 0
(2) inserted into (1) L L
du dv du L
with σ = E ------ gives the weak form EA ∫ ------ ------ dx = [ v ( σA ) ] + Aρg ∫ vdx
dx dx dx 0
0 0

du
3.5(b) Displacement interpolation: u = Nd e ------ = dN
------- de = Bd e
dx dx
T
dv
------ = b Te dN
T T T
Weight function: v = Nb e = be N ---------- = b Te B
dx dx
Inserted into the L
L
L
T T T T T
weak form gives: be EA ∫ B Bdx d e = be [ N ( σA ) ] 0 + Aρg ∫ N dx
0 0
⎧⎪ ⎨ ⎪ ⎩ ⎧⎪ ⎪ ⎪ ⎪ ⎨ ⎪ ⎪ ⎪ ⎪ ⎩
ke fe
T
but be is arbitrary ⇒ k e de = fe

3.5(c)
Diskretization: L1 L2 L3 Element lengths: L1 = L2 = L3 = L
D1 D2 D3 D4 Boundary conditions: D1 = D4 = 0
Element matrices: (reaction forces: R1 & R4)
1 1
EA AρgL
k e = EA ∫ B BLdξ = ------- 1 – 1 fe = Aρg ∫ N Ldξ = -------------- 1
T T
Truss element:
L –1 1 2 1
0 0

EA
Spring element: ke = k 1 – 1 where k = 2 -------
–1 1 L

1 –1 0 0 R1 1
EA AρgL
Assembly: K = ------- – 1 2 –1 0 F = P + -------------- 2
L 0 –1 3 –2 0 2 1
0 0 –2 2 R4 0
2
EA D2 D2 PL AρgL
Eqs. (2) & (3): ------- 2 – 1 = P + AρgL ⇒ = ----------- 3 + ---------------- 7
L –1 3 D AρgL ⁄ 2 D3 5EA 1 10E 4
3

– 3.9 (15) –
FEM for Engineering Applications—Exercises with Solutions / August 2008 / J. Faleskog

3.6(a)
L

Weighted residual: ∫
v [ ( EIw″ )″ + k z w – q ]dx = 0 (1)
–L
L L
L
Integration by parts: ∫
v [ ( EIw″ )″ ]dx = [ v ( EIw″ )′ ] – ∫
v′ [ ( EIw″ )′ ]dx =
–L
–L –L L
L L
= [ v ( EIw″ )′ ] – [ v′ ( EIw″ ) ] + ∫
v″EIw″dx (2)
–L –L
–L
(2) inserted into (1) with T = – ( EIw″ )′ and M = – EIw″ gives the weak form:
L L L
L L

v″EIw″dx + ∫
vkz wdx = [ vT ] – [ v′M ] + ∫
vqdx
–L –L
–L –L –L

2 2
d---------
w d N
3.6(b) Displacement interpolation: w = Nde 2
= ---------2- d e = Bd e
dx dx
2 2 T
T T d-------v- T d N T
Weight function: v = Nb e = be N = b e
------------ = b Te B
2 2
dx dx
L L L
T L
T T T T T dN T L
be ∫
B EIB dx + ∫
N k z N dx d e = be ∫
N q dx + [ N T ]
– ---------- M
–L dx –L
–L –L –L
⎧⎪ ⎪ ⎪ ⎪ ⎨ ⎪ ⎪ ⎪ ⎪ ⎩ ⎧⎪ ⎪ ⎪ ⎪ ⎪ ⎪ ⎨ ⎪ ⎪ ⎪ ⎪ ⎪ ⎪ ⎩
ke fe
T
but b e is arbitrary ⇒ ke d e = f e

3.6(c)

Stiffness 3 3L – 3 3L 78 22L 27 – 13L


L L
2 2 2 2
T λEI EI λEI
K = ∫ B EIBdx + ∫ N ---------Ndx = --------- 3L 4L – 3L L + --------------- 22L 8L 13L – 6L
T
matrix 4 3 3
L 2L – 3 – 3L 3 – 3L 105L 27 13L 78 – 22L
–L –L
2 2 2 2
3L L – 3L 4L – 13L – 6L – 22L 8L
R1
Load Reaction ⎛ 2EI 8λEI
R2 Eq. (4): --------- + -------------⎞ d 4 = M 0
vector ⎝ L 105L⎠
F = forces/moments
R3
105 M0 L
M0 ⇒ w′ ( L ) = d 4 = -------------------------- -----------
( 210 + 8λ ) EI

– 3.10 (15) –
FEM for Engineering Applications—Exercises with Solutions / August 2008 / J. Faleskog

3.6(d)

Element Beam (dof 1 to 4) Tensile & torsion springs


3 3L – 3 3L
stiffness L
2 2
(only dof 3 and 4)
EI
K beam = ∫ B EIBdx = --------- 3L 4L – 3L 2L
T
matrices 3EI EI
3
2L – 3 – 3L 3 – 3L k w = --------3- k θ = ------
–L
2 2 2L L
3L 2L – 3L 4L

The total stiffness matrix is obtained by assembly 3 3L – 3 3L


22
EI
of the stiffnesses from the beam and the springs: K = --------3- 3L 4L – 3L 2L
2L – 3 – 3L 6 – 3L
2 2
Force vector: F = F b + Fs 3L 2L – 3L 6L

1 R1 Reaction
L
force & moment
where Fb = ------ ∫ N dx = ---- L ⁄ 3
Q Q
Fs = R 2
T
2L 2 1 External point
–L
0
–L ⁄ 3 force & moment
0
Displacement boundary conditions: w1 = θ1 = 0. The reduced equation system becomes

3
EI- 6 – 3L w 2 Q w2 QL
--------
3
= ---- 1 ⇒ = ------------ 5L
2L – 3L 6L θ 2
2 2 –L ⁄ 3 θ2 27EI 1

3.7(a)
L
Weighted residual: ∫
v [ ( EIw″ )″ + ρgA ]dx = 0 (1)
–L
L L
L
Integration by parts: ∫
v [ ( EIw″ )″ ]dx = [ v ( EAw″ )′ ] – ∫
v′ [ ( EIw″ )′ ]dx =
–L
–L –L
L
L L
= [ v ( EAw″ )′ ] – [ v′ ( EAw″ ) ] + ∫
v″EIw″dx (2)
–L –L
–L
(2) inserted into (1) with T = –( EIw″ )′ and M = – EIw″ gives the weak form:
L L
L L

v″EIw″dx = [ vT ] – [ v′M ] – ρgA ∫ vdx
–L –L
–L –L
2 2
d---------
w d N
3.7(b) Displacement interpolation: w = Nde 2
= ---------2- d e = Bd e
dx dx
2 2 T
T T d-------v- T d N T T
Weight function: v = Nb e = be N = b e
------------ = b e B
2 2
dx dx
L L T
T L but be
T T T T dN
L T
be ∫
B EIB dx d e = be [ N T ] – ---------- M – ρgA ∫ N dx
–L dx –L is arbitrary
–L –L
⎧⎪ ⎨ ⎪ ⎩ ⎧⎪ ⎪ ⎪ ⎪ ⎪ ⎪ ⎨ ⎪ ⎪ ⎪ ⎪ ⎪ ⎪ ⎩ ⇒ k e d e = fe
ke fe

– 3.11 (15) –
FEM for Engineering Applications—Exercises with Solutions / August 2008 / J. Faleskog

3.7(c)

Element Beam (d.o.f 1 to 4) Spring (only d.o.f. 3)


3 3L – 3 3L
stiffness L
2 2 EI
EI
K balk = ∫ B EIBdx = --------- 3L 4L – 3L 2L
T K fjäder = k = --------3-
matrices: 3 2L
2L – 3 – 3L 3 – 3L
–L
2 2
3L 2L – 3L 4L

Assembly of total stiffness matrix 3 3L – 3 3L


22
EI
(the spring only contributes to d.o.f. 3): K = K balk + K fjäder = --------3- 3L 4L – 3L 2L
2L – 3 – 3L 4 – 3L
2 2
Force vector: F = F distributed + Fpoint 3L 2L – 3L 4L

1 R1 Reaktion
L
force/moment
where Fdistributed = – ρgA ∫ N dx = – ρgAL L ⁄ 3 F point = R 2
T

1 –P External point
–L
–L ⁄ 3 force/moment
0
P = ρgAL inserted gives:

R 1 – ρgAL Displacement boundary conditions: w1 = θ1 = 0


2 Reduced equation system (Eq. (3) & (4)):
F = R2 – ρgAL ⁄ 3
3
– 2ρgAL EI 4 – 3L w 2 w2 ρgAL
---------
3
= ρgAL – 2 ⇒ = – ---------------- 2
2 2L – 3L 4L θ 2
2
L⁄3 θ2 EI 4 ⁄ 3
ρgAL ⁄ 3
3.8 (a)
L
x Q
Weighted residual: ∫
v ( EIw″ )″ + ⎛⎝ 1 + ---⎞⎠ ------ dx = 0 (1)
L 2L
–L L L
L
Integration by parts: ∫
v [ ( EIw″ )″ ]dx = [ v ( EAw″ )′ ] – ∫
v′ [ ( EIw″ )′ ]dx =
–L
–L –L L
L L
= [ v ( EAw″ )′ ] – [ v′ ( EAw″ ) ] + ∫
v″EIw″dx (2)
–L –L
–L
(2) inserted into (1) with T = – ( EIw″ )′ and M = – EIw″ gives the weak form:
L L
L L 1 x dx

v″EIw″dx = [ vT ] – [ v′M ] – Q ∫ v --- ⎛⎝ 1 + ---⎞⎠ ------
–L –L 2 L L
–L –L
2 2
d---------
w d N
3.8 (b) Displacement interpolation: w = Nde 2
= ---------2- d e = Bd e
dx dx
2 2 T
T T d-------v- T d N T
Weight function: v = Nb e = b e N = b e
------------ = b Te B
2 2
dx dx
Inserted into the weak form gives
L L T
T L but b e
T T T T LdN T1⎛ x ⎞ dx
be ∫
B EIB dx d e = be [ N T ] – ---------- M – Q ∫ N --- 1 + --- ------
–L dx –L 2⎝ L⎠ L is arbitrary
–L –L
⎧⎪ ⎨ ⎪ ⎩ ⎧⎪ ⎪ ⎪ ⎪ ⎪ ⎪ ⎪ ⎪ ⎨ ⎪ ⎪ ⎪ ⎪ ⎪ ⎪ ⎪ ⎪ ⎩ ⇒k d = f
e e e
ke fe

– 3.12 (15) –
FEM for Engineering Applications—Exercises with Solutions / August 2008 / J. Faleskog

3.8 (c)
d1 d3
2L, EI
FEM, discretization:
(one element) d2 d4
ξ
−1 0 1 3 3L – 3 3L
L
2 2
EI
K = ∫ B EIBdx = --------3- 3L 4L – 3L 2L
T
Element stiffness matrix:
2L – 3 – 3L 3 – 3L
–L
2 2
3L 2L – 3L 4L

Displacement boundary conditions: d2 = d3 = d4 = 0 => Reaction forces


Nodal force vector: F = Fb + F point –P External point force
L 9
T1 x dx Q R2
where F b = –Q ∫ N --- ⎛ 1 + ---⎞ ------ = – ------ 4L Fpoint = Reaction
2 ⎝ L L⎠ 30 21 R3
–L force/moment
– 6L R4
3 3
EI- 3Q 2 PL 1 QL
Reduced equation system, Eq. (1): -------- 3d 1 = – P – ------- ⇒ d 1 = – --- --------- – --- ----------
3 10 3 EI 5 EI
2L
The deflection of the beam is obtained by the displacement interpolation (approx.) as:
3 3 3
2 PL 1 QL 2 – 3ξ + ξ x
w ( ξ ) = N 1 ( ξ )d 1 = – ⎛⎝ --- --------- + --- ----------⎞⎠ ⎛ ---------------------------⎞ , where ξ = ---
3 EI 5 EI ⎝ 4 ⎠ L

3.9(a)
3L ⁄ 2
d dT
Weighted residual: ∫
v ------ ⎛ kA ------⎞ + qA – hP ( T – T ∞ ) dx = 0 (1)
dx ⎝ dx ⎠
0
3L ⁄ 2 3L ⁄ 2
d dT dT 3L ⁄ 2 dv
Integration by parts: ∫ v ------ ⎛⎝ kA ------⎞⎠ dx = v ⎛⎝ kA ------⎞⎠ – ∫
------ ⎛ kA dT
------⎞ dx (2)
dx dx dx 0 dx ⎝ dx ⎠
0 dT 0
(2) inserted into (1) with – Q = kA ------ gives the weak form:
dx
3L ⁄ 2 3L ⁄ 2 3L ⁄ 2
dv 3L ⁄ 2

------ kA dT
------ dx + ∫
vhPTdx = [ v ( – Q ) ] + ∫
v ( qA + hpT ∞ )dx
dx dx 0
0 0 0
dT- dN
3.9 (b) Temperature interpolation: T = NT e ----- = ------- T e = BT e
dx dx
T
dv
------ = b e dN
T T T T T
Weight function: v = Nb e = be N ---------- = b e B
dx dx
Inserted into the weak form gives:
x2 x2 x2
T T T T T T x2
be ∫
B kAB dx + ∫ N hPN dx T e = be ∫
N ( qA + hpT ∞ ) dx + [ N ( – Q ) ]
x1
x1 x1 x1
⎧⎪ ⎪ ⎪ ⎪ ⎨ ⎪ ⎪ ⎪ ⎪ ⎩ ⎧⎪ ⎪ ⎪ ⎪ ⎪ ⎪ ⎨ ⎪ ⎪ ⎪ ⎪ ⎪ ⎪ ⎩
ke fe
T
but b e is arbitrary ⇒ ke T e = f e

– 3.13 (15) –
FEM for Engineering Applications—Exercises with Solutions / August 2008 / J. Faleskog

3.9 (c)
1 e1 2 e2 3 e3 4
FEM - model:
T1 T2 T3 T4
For one element applies:
1 1
⎧ kA ⎫
k e = ∫ ---------- – 1 kA ---------- – 1 1 --- dξ + ∫ 1 – ξ hP 1 – ξ ξ --- dξ =
1 1 L L kA
⎨ hPL = 12 ------ ⎬ = ------ 4 – 1
L⁄2 1 L⁄2 2 ξ 2 ⎩
L ⎭ L –1 4
0 0
1
⎧ kA ⎫
fe = ∫ 1 – ξ ( qA + hPT∞ ) --- dξ =
L kA
⎨q = 0 ; hPL = 12 ------ ⎬ = 3 ------ T ∞ 1
ξ 2 ⎩
L ⎭ L 1
0
4 –1 0 0 1 Q R1
kA – 1 8 – 1 0 kA 2 + 0
Assembly of system matrix gives K = ------
L 0 – 1 8 –1
and the r.h.s F = 3 ------ T ∞
L 2 0
Equation system: 0 0 –1 4 1 – Q R4
4T ∞ Q R1 4 Q R1 L ⁄ ( kAT ∞ )
4 –1 0 0 1 4 –1 0 0 3
kA – 1 8 –1 0 T2 kA 2 + 0 ⇔ –1 8 –1 0 T ⁄ T ∞
------ = 3 ------T ∞ 2
= 6 + 0
L 0 –1 8 –1 T3 L 2 0 0 –1 8 –1 T3 ⁄ T ∞ 6 0
0 0 –1 4 T∞ 1 – Q R4 0 0 –1 4 1 3 – Q R4 ⁄ ( kAT ∞ )
L

Reduced Eq. system (2) & (3): “Reaction flux”

8 –1 T2 ⁄ T∞ = 6 – { –1 ⋅ 4 + 0 ⋅ 1 } ⇒
T2 T∞
= ------ 29 = T ∞ 1.3809
–1 8 T3 ⁄ T∞ 6 – {0 ⋅ 4 – 1 ⋅ 1 } T3 21 22 1.0476

3.9 (d)
T ξ2 0 –Q1
Convection at x=3L/2 gives for element 3: [ N ( –Q ) ]ξ1 c = –
–Q 2 0
Consider only the contribution from element node 2, since the contribution from
the element node 1 is cancelled by the contribution from element node 2 in element 2,
furthermore use that Q 2 = hA ( T2 – T ∞ )
· ξ
where hA = h ( PL ⁄ 96 ) = kA ⁄ ( 8L ) T kA 0 0 T 1 kA
⇒ [ N ( – Q ) ] ξ = ------ + ------
0 2
1
8L 0 1 T 8L T ∞
2
The equation system is modified according to:
4 –1 0 0 4 3 Q R1 L ⁄ ( kAT ∞ ) T2 1.3811
–1 8 –1 0 T2 ⁄ T∞ 6
= + 0 ⇒ T 3 = T ∞ 1.0491
0 – 1 8 –1 T3 ⁄ T∞ 6
0
1 1 T4 1.0119
8 T4 ⁄ T∞
0 0 –1 4 + --
- 3 + --- 0
8
3.10 (a)
x2
d dT
Weighted residual: ∫
v ------ ⎛⎝ kA ------⎞⎠ – hP ( T – T ∞ ) dx = 0 (1)
dx dx
x1
x2 x2
x2
d dT dT dv dT
Integration by parts: ∫ v ------ ⎛⎝ kA ------⎞⎠ dx = v ⎛⎝ kA ------⎞⎠ – ∫ ------ ⎛ kA ------⎞ dx (2)
dx dx dx x1 dx ⎝ dx ⎠
x1 1 x
dT
(2) inserted into (1) with – Q = kA -----
- gives the weak form:
x2 dxx2 x2
dv dT x2
------ kA ------ dx + ∫ vhPTdx = [ v ( – Q ) ]x + ∫ vhpT∞ dx
∫ dx dx 1

x1 x1 x1

– 3.14 (15) –
FEM for Engineering Applications—Exercises with Solutions / August 2008 / J. Faleskog

dT-
----- dN
3.10 (b) Temperature interpolation: T = NT e = ------- T e = BT e
dx dx
T
dv
------ = b e dN
T T T T T
Weight function: v = Nb e = b e N ---------- = b e B
dx dx
Inserted into the weak form gives:
x2 x2 x2
T T T T T T x2
be ∫
B kAB dx + ∫ N hPN dx T e = be ∫
N hpT ∞ dx + [ N ( – Q ) ]
x1
x1 x1 x1
⎧⎪ ⎪ ⎪ ⎪ ⎨ ⎪ ⎪ ⎪ ⎪ ⎩ ⎧⎪ ⎪ ⎪ ⎪ ⎪ ⎨ ⎪ ⎪ ⎪ ⎪ ⎪ ⎩
ke fe
T
but be is arbitrary ⇒ ke T e = fe

3.10 (c)
1 e1 2 e2 3
FEM - model:
T1 T2 T3
For one element applies:
1 1
⎧ kA ⎫
k e = ∫ ---------- – 1 kA ---------- – 1 1 --- dξ + ∫ 1 – ξ hP 1 – ξ ξ --- dξ =
1 1 L L 2kA
⎨ hPL = 16 ------ ⎬ = ---------- 7 – 1
L⁄2 1 L⁄2 2 ξ 2 ⎩
L ⎭ 3L – 1 7
0 0
1
hPLT ∞
fe = ∫ 1 – ξ hPT ∞ --- dξ = ----------------- 1
L
ξ 2 4 1
0
Assembly of the stiffness matrix gives: 1
2 kA
7 –1 0 hPLT ∞ 1 0
System matrix: K = --- ------ – 1 14 – 1
3L
and r.h.s. F = ----------------- 2 + hPLT ∞
4 –Q R
=
0 –1 7 1 -----------------
hPLT ∞

5 20
hPLT ∞ 2 ⎧ kA ⎛ hPL ⎞ T ∞ ⎫ kA 8
⇒ F = ----------------- = ⎨ ------ ⎝ --------------⎠ ------ ⎬ = ------ T∞
4 QR ⎩
L kA ⁄ L 4 ⎭ L 4Q R
1– ----------------- 4– -----------------
hPLT ∞ hPLT ∞
Equation system: = 16

T1 20 30
7 –1 0 7 –1 0 T1
2 kA kA 8 12
--- ------ – 1 14 – 1 T2 = ------ T ∞ ⇔ – 1 14 – 1 T 2 = T∞
3L L 4Q R 6Q R
0 – 1 7 T 3 = T∞ 4 – ----------------- 0 –1 7 T∞ 6 – -----------------
hPLT ∞ hPLT ∞

7 – 1 T 1 = 30 – ( 0 ) T ⇒ T 1 = -----
1- 433
T ≈ 4.46 T ∞
Eq. (1) & (2) gives: ∞ 97 121 ∞
– 1 14 T 2 12 – ( – 1 ) T2 1.25

– 3.15 (15) –
FEM for Engineering Applications—Exercises with Solutions / August 2008 / J. Faleskog

4. FEM: trusses and beams

4.1 A uniaxial bar is modelled by a linear truss element.


x=0 x=L
For a certain applied load, the node displacement x
shown in the figure results. (a) Show that the strain
developing in the element is ε ( x ) = ε 0 and (b) show
u1 = δ 0 u 2 = δ0 + ε 0 L
that if ε0 = 0, the element is subjected to a rigid body
motion equal to δ0.

4.2 Derive the four shape functions for


the uniaxial “cubic” element shown to Nod 1 Nod 3 Nod 4 Nod 2
the right. Express the shape functions ξ
using the natural coordinate ξ. ξ = −1 ξ = −1/3 ξ = 1/3 ξ=1

4.3 The figure to the right shows a uniaxial isoparamet-


ric element where a 2nd degree polynomial is used for
Nod 1 Nod 3 Nod 2
the interpolation of the displacement. The coordinates of x
the nodes can be seen in the figure, where λ is a non- −L λL L
dimensional parameter in the interval: – 1 < λ < 1 .
Assume that the node displacements {u1, u2, u3} are
known and calculate the strain in the element.
Hint: express the strain as a function of the natural coordinate ξ, see below.

Coordinates:
1 3 2
∑ ξ
3
x(ξ) = N k xk
k=1 −1 0 1
N 1 = – ( 1 – ξ )ξ ⁄ 2
1 3 2 Primary variable:
x N 2 = ( 1 + ξ )ξ ⁄ 2

3
x1 x3 x2 φ( ξ) = Nk φ k 2
k=1 N3 = 1 – ξ

K x = q0
P
4.4 The bar in the figure to the right is subjected to an
uniformly distributed axial load Kx = q0 and a point force E, A
x=0 x=L
P. Analyse the bar by use of the finite element method
with (a) one linear element and (b) two linear element. Exact solution:
2
Compare the solutions with the exact solution given Px q 0 L x 1 x 2
u ( x ) = ------- + ----------- ⎛ --- – --- ⎛ ---⎞ ⎞
below the figure. EA E ⎝ L 2 ⎝ L⎠ ⎠
P
σ ( x ) = --- + q 0 L ( 1 – x ⁄ L )
A

– 4.1 (6) –
FEM for Engineering Applications—Exercises with Solutions / August 2008 / J. Faleskog

4.5 Carry out a finite element analysis of the uniaxial bar problem shown in figure (a) below.
Divide the bar in two linear elements of the same length. A linear element with shape func-
tions is shown in figure (b) below.
N 1 L 2
K x = q0 ------3-
u1 u2
E, A m
ξ
0 1
x=0 3 x = 2L
x = --- L N 1 = 1 – ξ, N 2 = ξ dx = Ldξ
2
(a) (b)

4.6 One requirement the displacement θ2


interpolation in an element must satisfy, θ 1 = θ2 = θ
is that it should be able to model an arbi- θ
trary rigid body motion. For a plane 2- w1 = δ – θ L
θ w2
node beam element with 2 degrees of 1 δ w2 = δ + θ L
freedom at each node, this means that w1
x
the deflection of the beam must be able
to take the form −L 0 L
w(x ) = δ + θx ,
where δ and θ are parameters describing an arbitrary rigid body motion as illustrated in the
figure to the right. Show that the displacement interpolation of the element can satisfy a rigid
body motion as described above.

4.7 The figure to the left θ2 w1 = a – b – c


shows an initially straight θ0
1 0
δ θ 1 = b ⁄ L + 2c ⁄ L
beam element that is sub- w2
θ1 R 0 = -----
jected to a deformation state κ 0
w1 w2 = a + b – c
that results in a constant cur-
vature κ 0 = 1 ⁄ R0 , where R0 x θ 2 = b ⁄ L – 2c ⁄ L
is the radius of curvature. −L 0 L
Curvature is here defined as
w″ = – κ (small deformations is assumed). The displacements of the two nodes of the ele-
ment are shown in the figure. (a) Calculate the curvature κ 0 and (b) find the slope (angle θ0)
and the displacement δ0 at the midpoint of the element (x = 0).

q ( x ) [N/m] P
4.8 A cantilever beam is loaded by a point force P, a
moment M and a uniformly distributed force per unit
length q ( x ) = Q ⁄ ( 2L ) (Q is the total resultant M
x x = 2L
force) according to the figure to the right. The bend-
ing stiffness of the beam is EI and its length 2L. Ana-
lyse the beam with FEM and use a 2-node beam Exact solution:
element (3rd degree polynomial for the interpolation PL
3
x 2 x 3 ML x 2
2
w ( x ) = --------- ⎛ 6 ⎛ ---⎞ – ⎛ ---⎞ ⎞ + ----------- ⎛ ---⎞
of the deflection). Carry out the analysis using (a) 6EI ⎝ ⎝ L⎠ ⎝ L⎠ ⎠ 2EI ⎝ L⎠
one element and (b) two elements. Compare the 3
QL 1 x 2 1 x 3 1 x 4
results with the exact solution shown below the fig- + ---------- ⎛ --- ⎛ ---⎞ – --- ⎛ --- ⎞ + ------ ⎛ ---⎞ ⎞
EI ⎝ 2 ⎝ L⎠ 6 ⎝ L⎠ 48 ⎝ L⎠ ⎠
ure.

– 4.2 (6) –
FEM for Engineering Applications—Exercises with Solutions / August 2008 / J. Faleskog

4.9 The figure below shows a cantilever beam, which is subjected to a distributed force, a
moment and a point force. The bending stiffness of the beam is EI. The beam is during a FEM-
analysis modelled by two 2-nodes beam element. The coordinates of the three nodes used in
the FEM-model are: x = { 0, 2L, 3L } . Determine the force vector, where also the reaction
forces should be indicated.

Q x N-
q(x) = ------ ⋅ --- ---
z 2L L m
P

M x
x=0 x = 3L
x = 2L

FORMULAS

1D: φ1
φ ( ξ ) = N1 φ 1 + N2 φ 2 = N 1 N 2 N1 = 1 – ξ N2 = ξ
1 φ2
L 2
φ1 φ2 N
ξ L L

∫ ∫ dN
T
L N- 1
N Ndx = { dx = Ldξ } = --- 2 1 dx = --- 1 – 1
T
0 1 ---------- -----
6 1 2 dx dx L –1 1
0 0

2 2
Balkelement: d N 1d N
Deflection: w ( ξ ) = N 1 d 1 + N 2 d 2 + N 3 d 3 + N 4 d 4 = Nd e , B = ---------2- = -----2 ---------2-
dx L dξ
d1 d3 3 2 3
2L, EI N 1 = ( 2 – 3ξ + ξ ) ⁄ 4, N2 = L( 1 – ξ – ξ + ξ ) ⁄ 4
3 2 3
d2 d4 N 3 = ( 2 + 3ξ – ξ ) ⁄ 4, N4 = L( – 1 – ξ + ξ + ξ ) ⁄ 4
ξ
−1 0 1
3 3L – 3 3L 78 22L 27 – 13L
L L

∫ B Bdx = -------- ∫ N Ndx = --------


2 2 2 2
T 1 3L 4L – 3L 2L T L 22L 8L 13L – 6L
- -
3 105
2L – 3 – 3L 3 –3L 27 13L 78 – 22L
–L –L
2 2 2 2
3L 2L – 3L 4L – 13L – 6L – 22L 8L

1 1 1 –6
N dξ = L ⁄ 3
1
N ξdξ = ------ – L
T T
∫ ∫ 15 6
1
–1 –1
–L ⁄ 3 –L

– 4.3 (6) –
FEM for Engineering Applications—Exercises with Solutions / August 2008 / J. Faleskog

Solutions
4.1
Displacement in the element: u = N 1 u 1 + N 2 u 2 = δ 0 + ε 0 ξL = { x = ξL } = δ 0 + ε 0 x
du
(a) Strain in the element: ε ( x ) = ------ = ε0
dx
(b) The case ε0 = 0 results in the rigid body motion since u ( x ) = δ0

4.2 Use Lagrange interpolation:


N 1 = ( 1 – ξ ) ( 1 – 3ξ ) ( 1 + 3ξ ) ⁄ C 1 N 1(– 1) = 1 ⇒ C = –16 1

N 2 = ( 1 + ξ ) ( 1 – 3ξ ) ( 1 + 3ξ ) ⁄ C 2 N (1) = 1 ⇒ C = – 16
2 2

N 3 = ( 1 + ξ ) ( 1 – ξ ) ( 1 – 3ξ ) ⁄ C 3 N (– 1 ⁄ 3) = 1 ⇒ C = 16 ⁄ 9
3 3

N 4 = ( 1 + ξ ) ( 1 – ξ ) ( 1 + 3ξ ) ⁄ C 4 N (1 ⁄ 3) = 1 ⇒ C = 16 ⁄ 9
4 4
4.3
Strain: du du dξ
ε = ------ = ------ ------ ⎫⎪
dx dξ dx
⎪⎬ ⇒ ε = u 2 – u1 ( u1 + u 2 – 2u 3 ) 1
+ ------------------------------------- ξ -----------------------
--------- x dξ = L ( 1 – 2λξ )dξ ⎪
3 ----------------
∂N 2L L ( 1 – 2λξ )
⎪⎭
k
dx = ∑ ∂ξ k
k=1 Note! singular for 2λξ = 1
4.4
(a) One element solution, discretization: D1 D2
Element stiffness matrix: Uniform load contribution to the nodal force vector:
1 1 ALq
f b = ∫ N K x ALdξ = ALq0 ∫ 1 – ξ dξ = ------------0- 1
EA
k e = ------- 1 – 1
T
L –1 1 ξ 2 1
Boundary conditions 0 0
Reaction force
2
PL- q----------
0L
Eq. EA D = 0 ALq Eq. 2 gives: D 2 = ------ + -
system: ------- 1 –1 1
= R + ------------- 1
0 EA 2E
L –1 1 D2 P 2 1
Eq. 1 then gives: R = – P – ALq 0
e1 e2
(b) Two element solution, discretization:
D1 D2 D3
Element stiffness matrix: Uniform load contribution to the nodal force vector:
1 1 ALq 0
= f b2 = ∫ N K x A --- dξ = ------- q 0 ∫ 1 – ξ dξ = ------------- 1
EA L AL
k1 = k 2 = ------- 1 –1
T
fb1
L –1 1 2 2 ξ 4 1
0 0

Eq. 2
1 –1 0 D1 = 0 R ALq 1 Eq. 2 & 3: D 2 PL- 1 q----------
L
syst.: 2EA
----------- – 1 2 – 1 = 0 + ------------0- 2 = ---------- + 0 - 3
D2 D 2EA 2 8E 4
L 4 3
0 –1 1 D3 P 1 Eq. 1 then gives: R = – P – ALq0
Note! The point force solution is exact and independent of the number of element used,
whereas the distributed load solution is approximate. The forces acting at the nodes
are in global equilibrium, i.e. external loads are in balance with internal (reaction) forces.

– 4.4 (6) –
FEM for Engineering Applications—Exercises with Solutions / August 2008 / J. Faleskog

4.5
e1 e2 Element stiff- k = k = EA
------- 1 – 1
ness matrices: 1 2 L –1 1
D1 D2 D3
1 1

Element 2: fQ =
T
N K x ALdξ = ALq 0 1 – ξ dξ = ALq
------------0- 1
∫ ∫ 8 3
ξ
0 1⁄2
B.C. Reaction forces

1 –1 0 D 1 = 0 ALq 0 0
R1

2
Assembly: EA q0L
------- – 1 2 –1 D2 = ------------- 1 + 0 D 2 = ----------
-
L 8 16E
0 –1 1 D 3 = 0 3 R3
EA ALq EA 3ALq 7ALq
Reaction forces: R1 = ------- ( – D 2 ) = – ------------0- R3 = ------- ( – D 2 ) – ---------------0- = – ---------------0-
L 16 L 8 16
2L
ALq 0
Note! – ( R1 + R 3 ) = ∫
K x Adx = -------------
2
3L ⁄ 2

4.6 4

w ( x ) = w ( ξL ) = ∑
N i di = N 1 ( δ – Lθ ) + N 2 θ + N 3 ( δ + Lθ ) + N 4 θ
i=1
= δ ( N 1 + N 3 ) + θ ( N 2 + N 4 + L ( N 3 – N 1 ) ) = δ + θξL = δ + θx
4.7
2
d Ni
4
N 1″ ⎛⎜ N 2″ N 3″ N 4″
- θ = 2c
⎞⎟
κ 0 = – w″ = – ∑ ----------
dx
i=1
2
- d i = – --------
L
2
⎝ L
2
- θ 1 + --------
- w 1 + --------
L
2
- w 2 + --------
L
2 2
------
L
2

4

δ0 = w ( x = 0 ) = ∑
N i ( ξ = 0 )d i = ( N 1 w 1 + N 2 θ 1 + N 3 w 2 + N 4 θ 2 ) = a
ξ=0
i=1

⎛⎝ ⎞⎠
4
dN i ( ξ = 0 ) N 1′ N 2′ N 3′ N 4′ b
θ 0 = w′ ( x = 0 ) = ∑
--------------------------- d i = ------- - θ 1 + -------
- w 1 + ------- -θ
- w 2 + ------- = ---
dx L L L L 2 L
ξ=0
i=1
4.8
D1 D3
(a) Discretization, one element:
D2 D4

Element stiffness matrix: Distributed load contribution


3 3L – 3 3L to the nodal force vector:
L
2 2
3
EI 1
B EIBdx = --------3- 3L 4L – 3L 2L
T TQ Q
ke = ∫ fb = ∫ N ------ Ldξ = ---- L
2L – 3 – 3L 3 – 3L 2L 6 3
–L –1
2 2
3L 2L – 3L 4L –L

– 4.5 (6) –
FEM for Engineering Applications—Exercises with Solutions / August 2008 / J. Faleskog

4.8 cont.

Eq. 3 3L – 3 3L D1 = 0 R 3
system: EI- 3L 4L – 3L 2L 2 2 D2 = 0 MR Q
--------
3
= + ---- L
2L – 3 – 3L 3 – 3L D3 P 6 3
2 2 M –L
3L 2L – 3L 4L D4
3 2 3
= --------- 8 ⁄ 3 + ----------- 2 + ---------- 3
D3 PL ML QL
Eq. 3 & 4 give:
D4 EI 2 ⁄ L EI 2 ⁄ L 3EI 1 ⁄ L

Eq. 1 & 2 then give the reaction forces: R = – P – Q M R = – 2PL – M – QL

D1 D3 D5
(b) Discretization, two elements:
D2 D4 D6
Element stiffness matrices: Distributed load contribution
3 3L ⁄ 2 –3 3L to the nodal force vector: 6
L⁄2

∫ 4EI 3L ⁄ 2 L 22 1
T – 3L ⁄ 2 L ⁄ 2 TQL Q
k1 = k2 = B EIBdx = --------
L
3
-
– 3 – 3L ⁄ 2 3 – 3L ⁄ 2 f b1 = fb2 = ∫ N ------ --- dξ = ------ L
–L ⁄ 2 2L 2 24 6
2 2
3L ⁄ 2 L ⁄ 2 – 3L ⁄ 2 L –1
–L
3 3L ⁄ 2 –3 3L 0 0 D1 = 0
Eq. 2 2
R 6
3L ⁄ 2 L – 3L ⁄ 2 L ⁄ 2 0 0 D2 = 0 MR
system: L
4EI – 3 – 3L ⁄ 2 6 0 –3 3L ⁄ 2 D3 Q 12
---------
3
= 0 +------
L 3L ⁄ 2 L ⁄ 2
2 2 2 24
0 2L – 3L ⁄ 2 L ⁄ 2 D 4 0 0
0 0 – 3 – 3L ⁄ 2 3 – 3L ⁄ 2 D5 P 6
2 2 M –L
0 0 3L ⁄ 2 L ⁄ 2 – 3L ⁄ 2 L D 6

Eq. 3 - 6 give: D3 Eq. 1 & 2 then give the reaction


5 1 17
3 2 3

D4 PL
= --------- 9 ⁄ L ML
+ ----------- 2 ⁄ L QL
+ ------------ 28 ⁄L forces: R = –P–Q
D5 6EI 16 2EI 4 48EI 48 M R = – 2PL – M – QL
D6 12 ⁄ L 4⁄L 32 ⁄ L

Note! The solutions for P and M are exact independent of the number of beam elements
used, whereas the distributed load solution is approximate. Also note that the forces
acting at the nodes are in global equilibrium, i.e. external loads are in balance
with internal (reaction) forces.
4.9
D1 D3 D5 B.C. & kinematical constraint: D1 = D2 = D3 = 0
e1 e2 (give reaction forces/moments: R1, R2 & R3)
D2 D4 D6 1

Element 1: x = L ( 1 + ξ ) ⇒ dx = Ldξ , fb = ∫
T
N qLdξ where
Q
q = ------ ( 1 + ξ )
2L
1 –1
3
(-------------------------------
2 – 3ξ + ξ )- ----- Q- 3Q 2QL 7Q QL
f b1 = ∫
( 1 + ξ )Ldξ = ------- f b2 = ----------- f b3 = ------- f b4 = – --------
4 2L 10 15 10 5
–1

⇒F T
= R + 3Q - R2 + 2QL
1 ------
10
----------- R3 + 7Q
15
------- M – QL
10
--------
5
–P 0

– 4.6 (6) –
FEM for Engineering Applications—Exercises with Solutions / August 2008 / J. Faleskog

5. FEM: planar frames of trusses and beams


y/L
5.1 The figure to the right shows a structure with three linear
elastic truss elements with elastic modulus E. Element 1 and 3 (0,1) (1,1)
have cross sectional area A and length L. Element 2 has cross
sectional area 2A and length 2L . The structure is sub- 3 2
jected to a point force Q and a force per unit volume (body
force) K x = Q ⁄ ( AL ) according to the figure. Calculate the Kx
displacement at node 2, all reaction forces and the normal x/L
stress in each one of the truss elements. 1 (1,0)
Q

5.2 A truss structure containing three trusses, all with elastic


L
modulus E and cross sectional area A, is shown to the right. The
structure is loaded by one point force Q/2 and a body force of
total magnitude Q acting on the vertical truss member down- Q L
wards. Model the structure by use of three linear elements and 2L
calculate the displacements and possible reaction forces at the
nodes. Note, the displacements at the nodes will in the current
case agree with the exact solution. Will the numerical solution Q⁄2
deviate from the exact one? If so, how?

5.3 Analyse the cantilever beam to the right by use of FEM.


Use a 2-node element, which allows for both axial deforma- 45o
tion and development of curvature (bending). The elastic mod-
ulus of the beam is E and the cross section is shown in the h
figure. Note that with the load applied in the present case, the h
FEM solution will agree with the exact solution. Especially, 2P
evaluate the solution for the case h ⁄ L = 1 ⁄ 10 .

5.4 A force per unit length q(x) is applied on a beam


q(x)
with elastic modulus E and a cross sectional area A y P
and a moment of inertia I. The left end of the beam
x
is clamped and the right end rests on an elastic sup- x = −2L
L k
port, here modelled by a vertical spring with spring
3
constant k = η EI ⁄ L .
x=2L
(a) Carry out a finite element analysis, where
the beam is modelled by one two-node element, and evaluate the deflection of the
beam. Here: η = 3 ⁄ 2 and q ( x ) = – Q ⁄ ( 2L ) ( x ⁄ L ) .
(b) Divide the beam into two element of equal to length and redo the analysis.
Note that the deflection at the nodes will in the current case always coincide with the exact
solution. The deflection between the nodes for 0 ≤ x ≤ 2L will deviate somewhat from the
exact solution due to distributed load q(x).

– 5.1 (6) –
FEM for Engineering Applications—Exercises with Solutions / August 2008 / J. Faleskog

5.5 Analyse the linear elastic planar frame work shown to the
L
right by use of FEM. Use a 2-node combined truss/beam ele-
ment allowing for both axial and bending deformation. The
cross section of the frame is displayed in the figure and the elas-
tic modulus is E. Will the FEM solution agree with the exact L
h
solution, i.e. with the Euler-Bernoulli beam theory, in the present h
case? P

5.6 The figure below shows a circular ring, which is an integral part of a flexible machine
member. The ring is subjected to diametrically opposed forces according to the figure. Deter-
mine the spring constant defined as k = P ⁄ δ by use of FEM. If the symmetry of the problem
is fully utilized, only a quarter of the ring needs to be modelled. The problem can for instance
be analysed by the Matlab based FEM program “frame2D”, available at the home page of the
course. If the displacement, δ, primarily is due to bending deformation (a good approximation
if R » h ), the spring constant of the ring can analytically be expressed as
4π EI
k = ------------------
2
- ------ ,
3
(π – 8)R
where E is the elastic modulus and I area moment of inertia. Note that in order for the FEM
solution to come close to this result, the FEM model requires that R » h .

δ⁄2 P y
“symmetric quarter”

R η P⁄2
R h δ⁄2

R
x
δ⁄2 P

– 5.2 (6) –
FEM for Engineering Applications—Exercises with Solutions / August 2008 / J. Faleskog

FORMULAS
Frames of truss/spring members (based on 2-node elements):
Global forulation for truss & spring elements
EA
y D4 Ke = k a –a spring constant for a truss element k = -------
–a a L
x 2 D3 2 c = cos φ
k φ where a = c sc
D2 2 s = sin φ
sc s
D1 2 l 12 = cos φ x = ( x 2 – x 1 ) ⁄ L
1 l12 l 12 m 12
alternativly a = m 12 = cos φ y = ( y 2 – y 1 ) ⁄ L
2
l12 m 12 m 12
Local/global transformation of nodal force vector: 2 2
L = ( x2 – x1 ) + ( y2 – y1 )

T l 12 m 12 0 0
Fe = T fe där T =
0 0 l 12 m 12

Frames of beam members (based on 2-node elements):


T
d e = u 1 w 1 θ 1y u 2 w 2 θ 2y
Local formulation
(local coordinate system) T
fe = f 1x f 1z M 1y f 2x f 2z M 2y
z w2, f2z
x EA
------- EA
0 0 – ------- 0 0
2L 2L
u2, f2x 3EI
---------
3EI
---------
3EI 3EI
– --------3- – --------2-
w1, f1z 0 3 2
0
2L 2L 2L 2L
θ2y, M2y
3EI
--------- 2EI-
-------- 3EI EI
u1, f1x 0 2
0 – --------2- ------
2L L 2L L
ke =
θ1y, M1y 2L EA
– ------- 0 0
EA
------- 0 0
2L 2L
3EI 3EI 3EI 3EI
0 – --------3- – --------2- 0 ---------
3
– --------2-
2L 2L 2L 2L
Global formulation 3EI EI 3EI 2EI
0 – --------2- ------ 0 – --------2- ---------
(global coordinate system) 2L L 2L L
D5, F5
z x
T
D2, F2 D4, F4 De = D1 D2 D3 D4 D5 D6
D6, F6
T
F e = F1 F2 F3 F4 F5 F 6
{X2, Z2}
Z D1, F1
D3, F3
{X1, Z1} Transformation scheme:
X d e = TD e ⎫ ⎫

⎬ ⇒ fe = k e TD e ⎪ T
fe = ke de ⎭ ⎬ ⇒ F e = T ke T D e = K e D e
T

F e = T fe ⎪

Transformation matrix:
X2 – X1
l x mx 0 l x = cos φxX = ------------------ = cos ϕ “Direction
T2 0 2a
T = där T2 = lz m z 0 cosines”
0 T2 Z2 – Z1
l z = cos φ zX = – ----------------- = – sin ϕ
0 0 1 2a ⎧ 2 2
Z2 – Z1 ⎪ lx + lz = 1
⇒⎨
m x = cos φ xZ = ----------------- = sin ϕ ⎪ m2 + m 2 = 1
2a
⎩ x z
X2 – X1
m z = cos φzZ = ------------------ = cos ϕ
2a

– 5.3 (6) –
FEM for Engineering Applications—Exercises with Solutions / August 2008 / J. Faleskog

Solutions
5.1
D6 D4 Element stiffness matrices:
D5 D3
1 0 –1 0 1 1 –1 –1 0 0 0 0
EA 0 0 0 0 k = EA EA 0 1
k 1 = ------- ------- 1 1 –1 –1 k 3 = ------- 0 –1
3 2 L –1 0 1 0
2 2L – 1 –1 1 1 L 0 0 0 0
D8 D2
0 0 0 0 –1 –1 1 1 0 –1 0 1
1 D1
D7 L
Q
Distributed load contribution to nodal force vector (el. 1): f b1 = ∫ NK x ALdx = – ---- 1
2 1
0
1
Q
= T f b1 = – ---- 0
T
Transformation to global coordinate system gives: F b1
2 1
with T = 1 0 0 0
00 1 0
Eq. D1 R1 – Q ⁄ 2 Bound. cond.: D1=D2=D3=D4=D5=D6=0 0
system: 2 0 –2 0
0 0 D2 R2 Give rise to the reaction forces R1 t.o.m. R6
0 0 0 0
D3 R3
1 1 –1 –1 Eq. 7 and 8 give: D7 QL
EA
0
1 1
0
–1 –1 D4 R4 = ----------- – 1
------- = D8 8EA – 5
2L 0 0 0 0 D5
0 0 R5
0 2 0 –2 D6 R6
-------, R 2 = 0, R 3 = 3Q
Eq. 1 - 6 then give: R 1 = 5Q -------
8 8
–2 0 –1 –1 0 0 3 1
D7 –Q ⁄ 2 3Q 5Q
0 0 –1 –1 0 –2 1 3 R 4 = -------, R 5 = 0, R 6 = -------
D8 –Q 8 8

The normal stress in one element: σ = EBde = EBTDe


D7 D7
D8 5Q
σ 1 = E – --1- --1- 1 0 0 0
D8 Q
= ------- σ 3 = E – --1- --1- 0 1 0 0 = -------
8A D7 L L 0 0 0 1 D5 8A
L L 0 0 1 0 D1 1 1
------- ------- 0 0
D2 1 1 2 2 D8 3Q D6
σ 2 = E – ---------- ---------- = -------
2L 2L 1 1 D3 8A
0 0 ------- -------
2 2 D4

5.2 D4 D6 Displacement B.C.: D2 = D3 = D4 = D5 = 0

D3 D5 ⎛ 1 + ---------
1 ⎞ 1 1
- – ---------- – 1 0 – ----------
1
----------
Eq. ⎝ 2 2⎠ 2 2 2 2 2 2
system: 1 1 1 1 D1 Q
– ---------- ---------- 0 0 ---------- – ---------- R2
2 2 2 2 2 2 2 2 0
D2 EA
------- –1 0 1 0 0 0 0 = Q ⁄ 2 + R3
L 0 0 0 1 0 –1 0 R4
D1
1 1 1 1 0 R5
– ---------- ---------- 0 0 ---------- – ----------
2 2 2 2 2 2 2 2 D6
0
1 1 1 1 ⎞
⎛ 1 + ---------
---------- – ---------- 0 – 1 – ---------- -
2 2 2 2 2 2 ⎝ 2 2⎠

D1 QL QL
⇒ = ----------- 3 – 2 = -------- 0.793 ⇒ R 2 R 3 R 4 R 5 = Q -0.207 -1.293 0.207 -0.207
D6 2EA EA -0.207
1– 2

The numerical solution results in a linear displacement variation in vertical element,


whereas the exact solution yields a quadratic displacement variation, due to the
distributed load.

– 5.4 (6) –
FEM for Engineering Applications—Exercises with Solutions / August 2008 / J. Faleskog

5.3
E, I, A, 2L D2 y D5
Equivalent P Discretization: x
D4
problem: h
4
2 D3
D1
D6
I = ------, A = h P
12
2
L 2
λ = --------- = 12 ⎛⎝ ---⎞⎠
AL
Eq. Here
λ 0 0 –λ 0 0 D1 = 0 Rx I h
system: Eq. 4 - 6 give: D 2PL ⁄ ( EA )
0 3 3L 0 – 3 3L D2 = 0 Rz 4 3 2⁄λ
PL
EI 2 D3 = 0 2 D 5 = --------- – 8 ⁄ 3 = – P ( 2L ) 3 ⁄ 3
--------- 0 3L 4L 0 – 3 L 2L = My EI
2L
3
– λ 0 0 λ 0 0 D4 D6 – 8 ⁄ ( 4L ) 2
– P ( 2L ) ⁄ 2
P
0 –3 – 3L 0 3 –3 L D5 P Eq. 1 - 3 then give the reaction forces:
2 2
0 3L 2L 0 – 3 L 4L D6 0 R x = – P, R y = P, M y = P2L

5.4
Q x
q ( x ) = – ------ ---
2L L
D2 y D5
EI, EA
One element solution: D4
D3 D1 x D6
D8
Eqn. EA EA D7
------- 0 0 – ------- 0 0 0 0
system: 2a 2a
R1
3EI- 3EI 3EI 3EI- 3EI
0 -------- --------- 0 – --------3- -------- 0 0 where a = 2L och k = --------
-
3
3 2 2 Q 2L
2a 2a 2a 2a 0 R 2 – ------
10
3EI 2EI 3EI 2EI 0
0 --------- --------- 0 – --------2- --------- 0 0 7QL
2 a a 0 R 3 – ----------- 0
2a 2a 60 D4
D4 QL
2 – 225 L P4L 1
K = – EA
------- 0 0
EA
------- 0 0 0 0 = P ⇒ D5 = ---------- ---------------
45 + ---------- 0
2a 2a D5 9Q EI EA
– ------- D6 1--- 0
3EI 3EI 3EI 3EI 10
0 – --------- – --------- 0 --------- + k – --------- 0 – k D 6 5
3 2 3 2 23QL
2a 2a 2a 2a 0 --------------
3EI 2EI 3EI 2EI 60
0 --------- --------- 0 --------- --------- 0 0 0
2 a 2 a 0
2a 2a
R8
0 0 0 0 0 0 0 0
0 0 0 0 –k 0 0 k

5.5
D2 y D5 Element stiffness λ 0 0 –λ 0 0
x 0 3 3L 0 – 3 3L
D4 matrices (local):
D1 el. 1 EI 2 2 A C
D3 D6 k e 1 = k e2 = --------3- 0 3L 4L 0 – 3 L 2L =
el. 2 2L – λ 0 0 λ 0 0 T
C B
Discreti- y 0 – 3 –3 L 0 3 –3 L
zation:
x 0 3L 2L
2
0 – 3 L 4L
2
Z
D8 2
L 2
λ = --------- = 12 ⎛ ---⎞
AL
here ⎝ h⎠
X D7 I
D9 T T T
T T2 0 A C T2 0 T 2 AT 2 T 2 CT 2
Element stiffness K e 1 = k e1 K e 2 = T k e1 T =
T T
=
T
0 T2 C B 0 T2 T T
matrix (globally): T 2 C T 2 T 2 BT 2

0 –1 0
Transformation matrix: T2 = 1 0 0
0 0 1

– 5.5 (6) –
FEM for Engineering Applications—Exercises with Solutions / August 2008 / J. Faleskog

5.5 cont.
Considering the displacement boundary conditions (D1=D2=D3=0)
the reduced equation system becomes:
3 + λ 0 3L – 3 0 3L D4 D4 2
0 ( h ⁄ L ) ⁄ 24
0 3 + λ – 3L 0 – λ 0 D5
0
D5
1
2 2 3
EI- 3L – 3L 8L – 3L 0 2L D6 D6 4PL 1⁄L
--------
3
= 0 ⇒ = -------------
2L –3 0 – 3L 3 0 – 3L D 7 P D7 EI 2
8 ⁄ 3 + ( h ⁄ L ) ⁄ 24
0 –λ 0 0 λ 0 D 0 D8 1
8
2 2 0
3L 0 2L – 3L 0 4L D9 D9 3 ⁄ ( 2L )

The reaction forces are given by Eqs. 1 - 3 as R1 = −P, R2 = 0, R3 = −2PL


The FEM solution coincides with the exact solution, since the frame only is subjected
to a point force.

5.6
y Ne = 4
The FEM analysis is carried out by use of a 2
node combined truss/beam element. To facilitate a
comparison with the analytical solution, the
2
geometry must be chosen such that ( h ⁄ R ) « 1 . Ne = 1
2
Since, h ∼ I ⁄ A , the cross section of the ring has Ne = 2 x
2
been choose such that R » I ⁄ A .
Ne = number of elements
1.3 1
k FEM – k analytic
-------------------------------------
k FEM ⁄ k analytic

k analytic

1.2 0.1 1
2
1.1 0.01

1.0
0 4 8 12 16 0.001
1 10 100
Number of elements
Number of elements

– 5.6 (6) –
FEM for Engineering Applications—Exercises with Solutions / August 2008 / J. Faleskog

6. FEM: 2D/3D solids

6.1 Derive the element stiffness matrix y/L N1 = 1 – ξ – η


for the CST element shown in the figure 1
to the right. Assume that the material is 3 N 2 = ξc N3 = η
isotropic, linear elastic with elastic mod-
ξ = x ⁄ L, η = y ⁄ L
ulus E and Poisson’s number ν = 0. 1 2 x/L
1

η
6.2 One way to satisfy compatibility across element
boundaries between regions of high to low order elements 1.0 3
is to use transition elements. A plane triangular transition
element is shown in Figure 4. The shape functions for the
vertex nodes of the element are displayed below the ele-
ment. Determine the shape function associated with node 4 1 4 2
and show that it fulfil standard requirements put on shape ξ
1.0
functions. N1 = 1 – ξ ( 3 – 2 ( ξ + η ) ) – η
N2 = ξ ( 2 ( ξ + η ) – 1 )
N3 = η

6.3 A plate containing a circular hole with radius R is mod-


elled by use of plane 8-noded bi-quadratic isoparametric y
elements. The figure to the right shows one such element
7
located at the hole of the plate. The element is symmetri- 4 3
cally located with respect to the y-axis and extends one 5
8 6
quarter of the circumference of the hole, i.e. the straight ele-
1 2
ment sides: 2-6-3 and 1-8-4, respectively, form 45o angles x
with respect to the y-axis. The nodes 1, 5 and 2 are placed at
the border of the hole. Determine the distance from the cen-
tre of the hole to the point { x 0, y 0 } , defined by the natural
coordinates ξ = ξ0 = 1 ⁄ 2, η = η0 = – 1 , i.e. calcu-
2 2
late ( x 0 + y 0 ) . How much does this point deviate from the geometric boundary (the radius)
of the circular hole?
The shape functions of the element are:
1 1
N 1 = – --- ( 1 – ξ ) ( 1 – η ) ( 1 + ξ + η ) , N 2 = – --- ( 1 + ξ ) ( 1 – η ) ( 1 – ξ + η ) ,
4 4

1 1
N 3 = – --- ( 1 + ξ ) ( 1 + η ) ( 1 – ξ – η ) , N 4 = – --- ( 1 – ξ ) ( 1 + η ) ( 1 + ξ – η ) ,
4 4

1 2 1 2 1 2 1 2
N 5 = --- ( 1 – ξ ) ( 1 – η ) , N 6 = --- ( 1 + ξ ) ( 1 – η ) , N 7 = --- ( 1 – ξ ) ( 1 + η ) , N 8 = --- ( 1 – ξ ) ( 1 – η ) .
2 2 2 2

– 6.1 (24) –
FEM for Engineering Applications—Exercises with Solutions / August 2008 / J. Faleskog

6.4 The Figure to the right shows a plane bi-linear isoparametric y


4-node element. The order of node numbering is opposite the L
one used in the natural coordinate system (ξ, η). As a conse- 1
quence, the determinant of the Jacobi matrix becomes negative.
The element stiffness matrix, which is calculated by area inte- 4 2 x
−L L
gration in the natural coordinate system, will then also become
3
negative. A FEM-analysis with such an element will “crash”.
Calculate the determinant of the Jacobi matrix for the element −L
with the erroneous node numbering to the right.
ζ
6.5 The figure to the right shows a three dimensional 11 14
20 node element of serendipity type. Assume that the 15 η
shape functions associated with the nodes 1 to 19 (N1, 19 10
12
..., N19) are known functions of the natural coordi- 16 13
9 18
nates: ξ, η, ζ. The element is shaped as a cube in the 20 3
7
natural coordinate system, defined in the interval −1 to 17 6
2
1 for each coordinate. Determine the shape function 4
5 ξ
associated with node 20, i.e. N20(ξ,η,ζ). 8
1

6.6 Figure (a) to the right shows a plane


y
(2D) isoparametric element with its shape 4 3
(a) b+l
functions given in figure (b) to the right.
(a) Determine the Jacobi matrix of the b 1 2
element, i.e x
a a + 3l
J = ∂x ⁄ ∂ξ ∂y ⁄ ∂ξ
∂x ⁄ ∂η ∂y ⁄ ∂η (b)
η N1 = ( 1 – ξ ) ( 1 – η ) ⁄ 4
4 3
(b) Determine the sub-matrix B1 in the 1
N2 = ( 1 + ξ)(1 – η ) ⁄ 4
B-matrix of the element
−1 1 ξ N3 = ( 1 + ξ ) ( 1 + η ) ⁄ 4
B = B1 B2 B3 B4 . −1
1 2 N4 = ( 1 – ξ) ( 1 + η) ⁄ 4

6.7 Model the plate (thickness h) shown to the right by use of a y/L
λ λ
4-noded bi-linear isoparametric element. Determine 1
(a) the coordinate transformation x ( ξ, η ) and y ( ξ, η ) , 4 3
(b) the Jacobi matrix J and its determinant J ,
(c) the strain vector ε (assume that the displacement vector de −1 1 x/L
is known),
(d) an expression for the element stiffness matrix ke.
1 λ
−1 λ 2

6.8 The plate in the above problem is loaded by a pressure p0


(uniform traction) acting on the side 1-4 and by its dead weight
(density ρ). The dead weight can be modelled as a force per unit ρ
volume (body force) K y = – ρg . Let λ = 1 ⁄ 2 and determine p g
0
the contributions to the nodal force vector from
(a) the pressure p0 and (b) the force per unit volume Ky.

– 6.2 (24) –
FEM for Engineering Applications—Exercises with Solutions / August 2008 / J. Faleskog

6.9 Calculate the contribution from the force per unit volume Ky to the nodal force vector in
the above problem by use of numerical integration based on Gauss-Legendre quadrature. Use:
(a) 1 × 1 and (b) 2 × 2 point integration scheme in the element.

6.10 A traction vector t (force per unit surface) is acting between points A and B located on
the edge of a plate of thickness h. The segment between A and B is straight and of length 2L.
Consider a linear variation of the traction vector according to

1 s 1 s
t = --- ⎛ 1 – ---⎞ tA + --- ⎛ 1 + ---⎞ tB ,
2⎝ L⎠ 2⎝ L⎠

where s is a natural coordinate, tA and tB are the traction vectors at the points A and B, respec-
tively, see the figure below. Determine the contribution to the total nodal force vector if the
plate is modelled by
(a) one isoparametric 4-node quadrilateral element,
(b) one isoparametric 8-node quadrilateral element, where the mid nodes are placed in the
middle between their corresponding corner nodes.
Assume that the traction vectors along AB are composed of a constant normal stress σ0 and a
constant shear stress τ0, such that

tx σ 0 cos θ – τ0 sin θ
tA = t B = = .
ty σ 0 sin θ + τ 0 cos θ

Use the results in (a) and (b) to evaluate the contribution to the total nodal force vector along
AB if the boundary is modelled by
(c) three equal isoparametric 4-node quadrilateral elements and
(d) three equal isoparametric 8-node quadrilateral elements, see the figure below.
θ
tb (a) (b) (c) (d)

s B 4
B B B B 7
L 3 3 6
8 3 5
4 4
6 4
0 2 3
ta 7 2
y 2 A 2 A 1 1
−L A 5
1 1 A A
x

6.11 A bi-linear rectangular element is y thickness h


loaded by its dead weight (gives rise to a Body force
body force), see the figure to the right. b 3
4 Kx = 0
Determine the nodal force vector. g
Assume that the acceleration of gravity,
1 2
x Ky = −ρ g
g, is known. a

– 6.3 (24) –
FEM for Engineering Applications—Exercises with Solutions / August 2008 / J. Faleskog

6.12 A triangular 2D domain is modelled by one plane triangular CST element according to
the figure below. The bottom side of the triangle is rigidly supported and the left side (x = 0) is
T
subjected to a linearly varying pressure, p(y), described by the traction vector t = [ p ( y ), 0 ] .
The material is isotropic, linear elastic with elastic modulus E and Poisson’s ratio ν = 1 ⁄ 3 .
Determine the node displacements and the reaction forces. The shape functions of the element
and the element stiffness matrix are given in the figure below.

y/L
(0,1) Element stiffness matrix:
p( y) = 4 2 –3 –1 –1 –1
y 3
p 0 ⎛ 1 – ---⎞ 2 4 –1 –1 –1 –3
⎝ L⎠
3Eh
K e = ---------- – 3 – 1 3 0 0 1
16 – 1 – 1 0 1 1 0
(1,0)
1 2
x/L –1 –1 0 1 1 0
–1 –3 1 0 0 3
Shape functions: Node displacement vector:
N 1 = 1 – ξ – η, N 2 = ξ, N3 = η T
d e = d 1x d1y d 2x d 2y d 3x d 3y
där ξ = x ⁄ L η = y⁄L

y/L
6.13 A CST element for 2D linear elastic analysis is shown
to the right. The material is isotropic, linear elastic with 1
elastic modulus E and Poisson’s ratio ν = 1/3. 3
(a) Show that the shape functions of the element is: 2
N 1 = 1 – ξ – η ; N 2 = 2ξ ; N 3 = η – ξ , where 1
ξ = x ⁄ L and η = y ⁄ L .
x/L
(b) Calculate the stresses in the element. The displace- 1/2
ment vector of the element, d e , is given in the figure, T
d e = 0 0 1--- --------
– 1- –-----1- Lε
0
where ε0 is a reference strain. 4 108 6
0

6.14 Consider a thin quadratic sheet metal of size y/l


l × l and thickness h of a linear elastic material (E, 4 3
ν). Model the sheet metal by use of two linear tri- e2
angular elements (CST-element) and carry out
FEM analyses for the three different load cases e1
x/l
(a), (b) and (c). Introduce appropriate displace- 1 2
ment boundary conditions, where symmetry con- (a) (b) (c)
ditions can be utilized, and determine node τ0
displacements and stresses in the elements. For
ρ
simplicity, let Poisson’s ratio be ν = 0 . g
Load cases: (a) uniaxial tension, σ0 σ0 τ0
(b) pure shear,
(c) dead weight, where ρ is the density and g acceleration of gravity.

– 6.4 (24) –
FEM for Engineering Applications—Exercises with Solutions / August 2008 / J. Faleskog

6.15 A rectangular sheet metal of thickness h is subjected to an uniaxial load corresponding to


a normal stress σ0. An exact analysis can for this case be carried out using a plane CST ele-
ment according to the FEM model shown in the figure below. Here, the uniaxial load is
applied as a traction vector, t, acting on the element side (edge) between node 2 and node 3.
The material is isotropic, linearly elastic with elastic modulus E and Poisson’s ratio ν = 1 ⁄ 3 .
The shape functions of the element and the element stiffness matrix Ke (plane stress) are given
in the figure.
(a) Calculate/evaluate the nodal force vector F, where also the reaction forces should be
marked. Use the coordinate s (see the figure) when calculating the consistent nodal
forces. Note that s = 0 at node 2 and s = 2L at node 3, which give the relations
x = L – s ⁄ 2 and y = s ⁄ 2 .
(b) Calculate the node displacements, de, and the reaction forces.
(c) Calculate the strains in the element and show that they agree with the exact solution, i.e.
εx = – νσ 0 ⁄ E , ε y = σ 0 ⁄ E and γ xy = 0 .
Shape functions:
σ0 y x y x y
N 1 = 1 – --- – ---, N 2 = ---, N 3 = ---
L L L L
1 0 Element stiffness matrix:
t = ------- 4 2 – 3 –1 –1 –1
L
3 2 σ0
2 4 – 1 –1 –1 –3
3Eh
L
s K e = ---------- – 3 – 1 3 0 0 1
16 – 1 – 1 0 1 1 0
1 2 x
–1 –1 0 1 1 0
–1 –3 1 0 0 3
σ0 Node displacement vector:
T
de = d 1x d 1y d2x d 2y d 3x d 3y

6.16 A rectangular sheet metal of thickness h is subjected to an uniaxial stress σ0. An exact
analysis of the problem can for instance be carried out by use of only one plane bi-linear 4-
node quadrilateral element as in the FEM model shown in the figure below. The position of the
element nodes are x ⁄ L = ± 1 and y ⁄ L = ± 1 . The uniaxial load is introduced in the model as
a traction vector applied on the element side (edge) between node 2 and node 3. The material
is isotropic, linearly elastic with elastic modulus E and Poisson’s ratio ν = 1 ⁄ 3 .
T
(a) Introduce the displacement vector D e = d 1x d 1y … d 4y and define the dis-
placement boundary conditions.
(b) Calculate/evaluate the nodal force vector Fe. Mark the reaction forces accord-
ing to f1x = R1x etc. (the shape functions of the element, isoparametric for-
mulation, are given in the figure below).
(c) Calculate all the reaction forces and also check that the nodal forces due to the
traction t agrees with the answer in (b) above. The element stiffness matrix
Ke and the resulting node displacement vector De are given below.

– 6.5 (24) –
FEM for Engineering Applications—Exercises with Solutions / August 2008 / J. Faleskog

y/L
4 3
2L
t = σ0 1
0
2L
x/L
σ0 σ0

1 2
Shape functions: 8 3 –5 0 –4 –3 1 0 0
η 3 8 0 1 –3 –4 0 –5 2 ⁄3
4 N1 = ( 1 – ξ ) ( 1 – η ) ⁄ 4 –5 0 8 –3 1 0 –4 3 2
3 Eh σ0 L 2 ⁄ 3
N2 = ( 1 + ξ ) ( 1 – η ) ⁄ 4 K e = ------- 0 1 –3 8 0 –5 3 –4 D e = ---------
16 – 4 E
ξ N3 = ( 1 + ξ ) ( 1 + η ) ⁄ 4
–3 1 0 8 3 –5 0 2
1 2 –3 –4 0 –5 3 8 0 1 0
N4 = ( 1 – ξ ) ( 1 + η ) ⁄ 4 1 0 –4 3 –5 0 8 –3 0
0 –5 3 –4 0 1 –3 8 0

6.17 Figure (a) to the right shows a plane 2L 2L


rectangular plate of thickness h, subjected (a)
to a pure bending moment. Utilizing the
symmetry in the problem, only half of the 2L
plate needs to be modelled. A crude FEM
model consisting only of one 4-node (bi-lin- M M
ear) quadrilateral element is shown in figure
(b). The coordinates of the nodes are evi- (b)
y/L
dent from the figure. The moment is
4 (2, 1) 3
replaced by an equivalent traction vector, t,
prescribed on the boundary x = 2L, as t = σ0 y ⁄ L
shown in the figure. The material is iso- 0
tropic, linear elastic with elastic modulus E
x/L
and Poisson’s ratio ν. Here plane stress con-
ditions are assumed to prevail.
(a) Calculate/evaluate the nodal force
vector F, where also the reaction 1 (2,−1) 2
forces should be indicated. T
D = D 1x D 1y D 2x D 2y D 3x D 3y D 4x D 4y
(b) Calculate the normal stress in the x-

direction as a function of position. = --- -----0- L 0 0 – 1 – 1 1 –1 0 0
Make use of the node displacement 3E
solution is given in figure (b).
6.18 A quadratic plate with edges of length 2L and of thickness h is loaded by its dead
weight and rotates around its diagonal with a constant angular velocity ω, see figure (a) below.
The lower corner of the plate is mounted on a bearing. The plate has density ρ and the material
is isotropic, linear elastic with elastic modulus E and Poisson’s ratio ν. Assume further that
plane stress conditions prevail and that ν = 0 . A rather coarse finite element model that to
some extent utilizes the symmetry of the problem is shown in figure (b) below. The model
consists of only one triangular element with a linear interpolation for the displacements (CST

– 6.6 (24) –
FEM for Engineering Applications—Exercises with Solutions / August 2008 / J. Faleskog

element). The inertia forces due to the angular velocity ω and the dead weight can be
2
addressed by introducing the body forces K x = ρω x and K y = – ρ g into the FEM analysis.
The shape functions Ni and the element stiffness matrix ke are given in the figure below.
(a) Calculate/evaluate the nodal force vector F, where also the reaction forces should be
included symbolically.
(b) Calculate all the displacements at the nodes de and the reaction forces.
(c) Calculate the stresses in the element.
ω y/L
(a) (b)
(0, 1)
2L 3

x/L
2
(1, 0)

2L
g 1
(0, −1)

Shape functions: Element stiffness matrix:


1 x y x
N 1 = --- ⎛⎝ 1 – --- – ---⎞⎠ , N 2 = --- 3 1 –4 –2 1 1
2 L L L 1 3 0 –2 –1 –1
1 x y Eh
N 3 = --- ⎛ 1 – --- + ---⎞ ke = ------- – 4 0 8 0 –4 0
2⎝ L L⎠ 8 –2 –2 0 4 2 –2
Nodal displacement vector: 1 –1 –4 2 3 –1
T 1 –1 0 –2 –1 3
d e = d 1x d 1y d 2x d 2y d 3x d 3y

6.19 A rectangular plate of thickness h rotates with a constant angular velocity ω, see Figure
(a) below. The material of the plate is isotropic linear elastic with elasticity modulus E, Pois-
son’s ratio ν and has density ρ. Plane stress is assumed to prevail in the plate and that ν = 0. If
the symmetry of the problem is considered and utilized, the plate can be modelled by only two
bi-linear isoparametric elements according to Figure (b) below. The node coordinates is evi-
dent from the figure. The inertia forces due to the angular velocity ω can be addressed by
2
introducing the body forces K x = ρω x and K y = 0 into the FEM analysis.

(a) Determine the contribution from the body force to the node force vector in element 2.
Hint: the coordinate transformation x = ( 3 + ξ )L , y = η L is useful in element 2.

(b) The resultant node displacements, DT, corresponding to the current load is given in the
figure below. Calculate the stresses at the three points: {x = 0, y = 0}, {x = L, y = 0}, {x
= 2L, y = 0}. The exact solution for the normal stress in the x-direction can be
2 2 2
expressed as σ xx ( x ) = ( ρω L ⁄ 2 ) ( 16 – ( x ⁄ L ) ) . How much does the FEM solution
deviates from the exact solution at the three points? At which point do the solutions
deviate the least?
Hint: the Jacobi matrix of the coordinate transformation is J = L I , where I is a unit

– 6.7 (24) –
FEM for Engineering Applications—Exercises with Solutions / August 2008 / J. Faleskog

matrix of dimension 2.
y/L
(a) ω (b)
5 4 (2, 1) 3 (4, 1)
(0, 1)
ρ, E, ν = 0
Elem.1 Elem. 2
x/L
4L
6 1 2
(0, -1)
T
D = D 1x D 1y D 2x … D 6x D 6y
2 3
4L 4 ρω L
= --- ---------------- 11 0 16 0 16 0 11 0 0 0 0 0
3 E

6.20 A plane solid of thickness h is subjected to a lin-


y
early varying pressure, p(x), acting on a segment of x
p ( x ) = p 0 ---
the boundary as shown in the figure to the right. The y = 2L L
3 6
material is isotropic, linear elastic with elastic modu- e4
lus E and Poisson’s ratio ν. The plane solid is mod- e3 e7
elled by triangular CST elements. The mesh is y = L 2 5 8
indicated in the figure, where also the displacement e2 e6
vector D and the nodal force vector F are shown. e1 e5
1 4 7 x
(a) Define the displacement boundary conditions. y=0
(b) Calculate/evaluate the nodal force vector. Mark x=0 x=L x = 2L
the presence of possible reaction forces as F1x T
D = D 1x D 1y D 2x D 2y D 8x D 8y
= R1x and so on.
(c) Calculate the stresses in element e1. Assume T
F = F 1x F 1y F 2x F 2y F 8x F 8y
plane stress prevail and that the displacement
vector D is known, where the zero displace-
ment boundary conditions may be enforced.
6.21 A cantilever beam is modelled by 20 plane 4-node quadrilateral elements arranged
according to the figure below, where the global numbering of nodes and elements are indi-
cated. All elements are rectangular, of equal size and of thickness h. The beam is made of a
linear elastic, isotropic material with elastic modulus E and Poisson’s ratio ν = 0. Plane stress
conditions is assumed to be valid. The left end of the beam is welded on a wall, which in the
present model is assumed to be rigid. The beam is subjected to a shearing load acting on its
right end as illustrated by the figure. The components of the displacement vector and the force
vector is also indicated in the figure.
(a) Define the displacement boundary conditions.
(b) Calculate the external load contributions to the node force vector, i.e. the contributions
from the traction vector.
(c) Calculate the stresses at the centroid of element e1. Assume that the displacement vec-

– 6.8 (24) –
FEM for Engineering Applications—Exercises with Solutions / August 2008 / J. Faleskog

tor D is known.
y
2a

1 e1 6 e5 11 e9 16 e13 21 e17 26 2b
2 e2 7 e6 12 e10 17 e14 22 e18 27 τ0
3 8 13 e11 18 23 e19 28
x
e3 e7 e15
4 e4 9 e8 14 e12 19 e16 24 e20 29 τ0
5 10 15 20 25 30

T T
D = D 1x D 1y D 2x D 2y D 30x D 30y F = F 1x F 1y F 2x F 2y F 30x F 30y

– 6.9 (24) –
FEM for Engineering Applications—Exercises with Solutions / August 2008 / J. Faleskog

FORMULAS
Plane (2D) triangular linear element: d 1x

d3y d 1y
y Displace- u ( x, y ) = N 1 0 N 2 0 N 3 0 d = Nd de =
d 2x
e e
d3x ments: v ( x, y ) 0 N1 0 N2 0 N3 d 2y
d 3x
d2y
1 d 3y
d1y Ae N 1 = --------- [ ( y 2 – y 3 ) ( x – x 2 ) + ( x 3 – x 2 ) ( y – y 2 ) ]
2A e
d2x
1
N 2 = --------- [ ( y 3 – y 1 ) ( x – x 3 ) + ( x 1 – x 3 ) ( y – y 3 ) ]
d1x 2A e
x 1
N 3 = --------- [ ( y 1 – y 2 ) ( x – x 1 ) + ( x 2 – x 1 ) ( y – y 1 ) ]
2A e

Strains: ε xx ∂N i ⁄ ∂x 0
ε yy = B de B = B1 B2 B3 Bi = 0 ∂N i ⁄ ∂y
γx y ∂N i ⁄ ∂y ∂N i ⁄ ∂x

Stresses: σ xx 1 ν 0 ε xx
E
σ yy = ------------------
-
2 ν 1 0 ε yy (Plane stress)
(1 – ν )
σ xy 0 0 ( 1 – ν ) ⁄ 2 γ xy

Plane (2D) quadrilateral bi-linear element:


4

y d
4y
d3y x = ∑N x i i
η
i
d3x 4 4 3
d1y
d4x y = ∑ Ni yi
ξ
i
d2y 1 2
d1x
N 1 = ( 1 – ξ ) ( 1 – η ) ⁄ 4, N 2 = ( 1 + ξ) ( 1 – η ) ⁄ 4
d2x
x N 3 = ( 1 + ξ ) ( 1 + η ) ⁄ 4, N4 = ( 1 – ξ ) ( 1 + η ) ⁄ 4

Displace- u ( ξ, η ) = N 1 0 N 2 0 N 3 0 N 4 0 d = Nd
ments: v ( ξ, η ) 0 N1 0 N2 0 N 3 0 N 4
e e

∂N i ⁄ ∂x 0
Deformation: B = B1 B 2 B 3 B 4 Bi = 0 ∂N i ⁄ ∂y
∂N i ⁄ ∂y ∂N i ⁄ ∂x

∂N i ⁄ ∂x –1 ∂N i ⁄ ∂ξ ∂x ⁄ ∂ξ ∂y ⁄ ∂ξ
where = J J =
∂N i ⁄ ∂y ∂N i ⁄ ∂η ∂x ⁄ ∂η ∂y ⁄ ∂η

Stresses: σ xx 1 ν 0 ε xx
E
σ yy = ------------------
2
- ν 1 0 ε yy (Plane stress)
(1 – ν )
σ xy 0 0 ( 1 – ν ) ⁄ 2 γ xy

– 6.10 (24) –
FEM for Engineering Applications—Exercises with Solutions / August 2008 / J. Faleskog

Solutions

–1 0 1 0 0 0 1 0 0
1
6.1 B = --- 0 – 1 0 0 0 1 ; plane stress with ν = 0 gives C = E 0 1 0
L
–1 –1 0 1 1 0 0 0 1⁄2

3 1 –2 –1 –1 0
3 0 –1 –1 –2
2
hL T T Eh 2 0 0 0
Element stiffness matrix: k e = ∫ B CBhdA = --------- B CB = -------
2 4 1 1 0
Ve
Sym. 1 0
2

6.2 Use that N 1 + N 2 + N 3 + N 4 = 1 ⇒ N 4 = 1 – ( N 1 + N 2 + N 3 ) = 4 ( 1 – ξ – η ) ξ


N 4 ( ξ, η ) must satisfy the condition:
(i) zero in nodes 1, 2 and 3, i.e. N 4 ( 0, 0 ) = N 4 ( 1, 0 ) = N 4 ( 0, 1 ) = 0
(ii) unity in node 4, i.e. N 4 ( 1 ⁄ 2, 0 ) = 1
Both conditions are satisfied!
6.3
2–1 2+1 1
For ξ 0 = 1 ⁄ 2, η 0 = – 1 we obtain N 1 = – ----------------, N 2 = ----------------, N 5 = --- , and
4 4 2
N3 = N4 = N6 = N7 = N8 = 0.
R R
Coordinates at the nodes: ( x 1, y 1 ) = ( – 1, 1 ) ------- , ( x 2, y 2 ) = ( 1, 1 ) ------- , ( x 5, y 5 ) = ( 0, 1 )R
2 2
R
Coord. in xy-plane: x 0 = x ( ξ0, η 0 ) = ∑ N i x i = --- ,
2
(1 + 2)
y 0 = y ( ξ 0, η 0 ) = ∑
N i y i = --------------------- R
2 2
x 0 + y 0 = R 5-------------------
2 2 + 2 2 ≈ 0.989R , i.e. the deviation of the FE-mesh
Distance from origin:
8 from the boundary of the hole is 1%.
6.4
The coordinates in the isoparametric element is given by the interpolation (coord.-transform):
4
L
x ( ξ, η ) = ∑
N i x i = L ( N 2 – N 4 ) = --- ( ξ – η )
2
i =4 1
L
y ( ξ, η ) = ∑
N i y i = L ( N 1 – N 3 ) = --- ( – ξ – η )
2
i=1
∂x L ∂x L ∂y L ∂y L
Partial derivatives: ------ = --- ; ------ = – --- ;------ = – --- ;------ = – ---
∂ξ 2 ∂η 2 ∂ξ 2 ∂η 2
2
L L
The Jacobi matrix and its determinant becomes: J = --- 1 – 1 ⇒ J = – -----
2 –1 – 1 2

– 6.11 (24) –
FEM for Engineering Applications—Exercises with Solutions / August 2008 / J. Faleskog

20 19

6.5 Use the property ∑


N i = 1 ⇒ N 20 = 1 – ∑
N i , Note that N1, ..., N19 are known!
i=1 i=1
Alternatively, derive the shape functions by use of the properties:
1 2
N20 = 0 in nodes 1 to 19 and that N20 = 1 ⇒ N 20 = --- ( 1 – ξ ) ( 1 – η ) ( 1 – ζ )
4

∂x- 1 3 ∂y
6.6 (a) ----- = --- [ ( 1 – η ) ( x 2 – x 1 ) + ( 1 + η ) ( x 3 – x 4 ) ] = --- l ; ------ = 0
∂ξ 4 2 ∂ξ
∂x ∂y 1 1 l
------ = 0 ; ------ = --- [ ( 1 – ξ ) ( y 4 – y 1 ) + ( 1 + ξ ) ( y 3 – y 2 ) ] = --- l ⇒J = --- 3 0
∂η ∂η 4 2 2 0 1

∂N 1 ⁄ ∂x ∂N 1 ⁄ ∂ξ
= --- 1 ⁄ 3 0 – ( 1 – η ) ⁄ 4 = – ----- ( 1 – η ) ⁄ 3
–1 2 1
(b) = J
∂N 1 ⁄ ∂y ∂N 1 ⁄ ∂η l 0 1 –( 1 – ξ ) ⁄ 4 2l ( 1 – ξ )

(1 – η) ⁄ 3 0
1
⇒ B1 = –"" ----- 0 (1 – ξ)
2l
(1 – ξ) (1 – η) ⁄ 3

6.7(a) x = ∑
N i x i = L ( 1 + λη )ξ, y= ∑
N i y i = Lη

J = ∂x ⁄ ∂ξ ∂y ⁄ ∂ξ = L 1 + λη 0 and J = L ( 1 + λη )
2
(b)
∂x ⁄ ∂η ∂y ⁄ ∂η λξ 1
(c)
Strain: ε = Bde, where the B-matrix is given by
∂N i ⁄ ∂x 0
∂N i ⁄ ∂x –1 ∂N i ⁄ ∂ξ
B = B 1 B 2 B 3 B 4 where B i = 0 ∂N i ⁄ ∂y with = J
∂N i ⁄ ∂y ∂N i ⁄ ∂η
∂N i ⁄ ∂y ∂N i ⁄ ∂x

= --- 1 ⁄ ( 1 + λη ) 0
Inverse of the Jacobi matrix becomes –1 1
J
L – λ ξ ⁄ ( 1 + λη ) 1
where
( 1 – η -)
–------------------- 1 – η-
∂N 1 ⁄ ∂x ∂N 2 ⁄ ∂x ---------------
1
= ------ 1 + λη 1
= ------ 1 + λη
∂N 1 ⁄ ∂y 4L ( 1 – η )λξ ∂N 2 ⁄ ∂y 4L ( 1 – η )λξ
– ( 1 – ξ ) + ------------------------ – ( 1 + ξ ) – ------------------------
1 + λη 1 + λη
1 + η- ( 1 + η )-
–--------------------
---------------
∂N 3 ⁄ ∂x 1 1 + λη ∂N 4 ⁄ ∂x 1 1 + λη
= ------ = ------
∂N 3 ⁄ ∂y 4L ( 1 + η )λξ ∂N 4 ⁄ ∂y 4L ( 1 + η )λξ
( 1 + ξ ) – ------------------------ ( 1 – ξ ) + ------------------------
1 + λη 1 + λη

– 6.12 (24) –
FEM for Engineering Applications—Exercises with Solutions / August 2008 / J. Faleskog

(d) T T T T
B 1 CB 1 B 1 CB 2 B 1 CB 3 B 1 CB 4
6.8 1 1 1 1 T T T T

= ∫------------------- , sin ∫ϕ ∫ = ∫∫
B 2 CB 1 B 2 CB 2 B 2 CB 3 B 2 CB 4
B CBhλJ dξdη =
T T 2
2 ke = 1
B CBhdA = hL ( 1 + λη )dξdη
14 = 2L 1 + λ , cos ϕ ------------------- T T T T
Ae 2 –1 –1 2 – 1 – 1 B 3 CB 1 B 3 CB 2 B 3 CB 3 B 3 CB 4
1+λ 1+λ T T T T
B 4 CB 1 B 4 CB 2 B 4 CB 3 B 4 CB 4
⎧ l14 ⎫ 1
T ⎪ dS = h ------ dη ⎪ T 2
= ∫
N tdS = ⎨ 2 ⎬ = ∫
N ξ = –1
thL 1 + λ dη

Se
⎩ ξ = – 1 ⎪⎭ –1

f 1x = f4x = p 0 hL, f 1y = f 4y = λp 0 hL
1 1
T
V = h J dξdη } = ∫ ∫
N f V h J dξdη
–1 – 1
2 1
J = L ( 1 + λη ) one obtains with λ = --- that
2
2 7 2
ρg, fb3y = f b4y = – --- h L ρg
6
2
05 05070 7 Note! f
∑ biy
= – 4hL ρg = – Ve ρg

6.9 Numerical integration with Gauss-Legendre quadrature


1 1 n ξ nη
2
f bIy = f ( ξ,
∫ ∫ I
η )dξdη = f (ξ ,
∑∑ I i
η j )w i w j where fI = N I ( ξ, η ) ( – ρg )hL ( 1 + λη )
–1 –1 i j

(a) 1 x 1 scheme: n ξ = n η = 1 , ξ 1 = η 1 = 0 and w 1 = 2 .


2 2
⇒ fb1y = N 1 ( 0, 0 ) ( – ρg )hL ( 2 ⋅ 2 ) = – h L ρg
2
in the same way we obtain f b2y = f b3y = f b4y = – h L ρg

–1 1
(b) 2 x 2 scheme: nξ = n η = 2 , ξ 1 = η 1 = ------- , ξ2 = η 2 = ------- and w 1 = w 2 = 1 .
3 3
2
⇒ fb1y = – ρghL [ N 1 ( ξ1, η 1 ) ( 1 + λη 1 ) ( 1 ⋅ 1 ) + N 1 ( ξ1, η 2 ) ( 1 + λη 2 ) ( 1 ⋅ 1 )
5 2
+ N 1 ( ξ 2, η 1 ) ( 1 + λη 1 ) ( 1 ⋅ 1 ) + N 1 ( ξ2, η 2 ) ( 1 + λη 2 ) ( 1 ⋅ 1 ) ] = – --- ρghL
6
5 2 7 2
in the same way we obtain f b2y = – --- ρghL and f b3y = f b4y = – --- h L ρg ,
6 6
i.e. the numerical integration are exact!

– 6.13 (24) –
FEM for Engineering Applications—Exercises with Solutions / August 2008 / J. Faleskog

6.10
T 2 2
Contributions to the nodal force vector: fs = ∫
N tdS , with dS = hds = h dx + dy .
Se
∂x ∂x ∂y ∂y
Here dx = ------ dξ + ------ dη and dy = ------ dξ + ------ dη , but ξ = 1 along 2-3 and thus
∂ξ ∂η ∂ξ ∂η
2 2
dξ = 0 , which give dS = h ( ∂x ⁄ ∂η ) + ( ∂y ⁄ ∂η ) dη .

(a) 4-node element: N 2 = ( 1 – η ) ⁄ 2 and N 3 = (1 + η) ⁄ 2


ξ=1 ξ=1
x = ( N 2 x2 + N3 x3 ) ⇒ ∂x ⁄ ∂η = ( x 3 – x 2 ) ⁄ 2 ,
ξ=1
y = ( N 2 y2 + N3 y3 ) ⇒ ∂y ⁄ ∂η = ( y 3 – y 2 ) ⁄ 2 ,
ξ=1

x 3 – x 2⎞ 2 y3 – y2 2
which give dS = h ⎛ ---------------
- + ⎛⎝ ----------------⎞⎠ dη = hLdη .
⎝ 2 ⎠ 2
1 1
1–η 1–η hL
f 2x = ∫
N 2 tx hLdη = ∫
N 2 ⎛⎝ ------------ t Ax + ------------ t Bx⎞⎠ hLdη = ------ ( 2t Ax + t Bx ) ,
2 2 3
–1 –1
hL hL hL
and f 2y = ------ ( 2t Ay + t By ) , f 3x = ------ ( t Ax + 2tBx ) , f 3y = ------ ( t Ay + 2t By ) .
3 3 3
( 1 – η ) ( –η ) ( 1 + η )η 2
(b) 8-node element: N 2 = ----------------------------- , N 3 = --------------------- , N 6 = 1–η
ξ=1 2 ξ=1 2 ξ=1

⎧ x2 + x3 ⎫ ⎛⎛ N
N6 N6
x = ( N 2 x2 + N3 x 3 + N 6 x6 ) = ⎨ x6 = ---------------- ⎬ = + ------⎞⎠ x 2 + ⎛⎝ N 3 + ------⎞⎠ x 3⎞
ξ=1 2 ⎭ ⎝⎝ 2 2 2 ⎠
⎩ ξ=1

⎧ y2 + y3 ⎫ ⎛⎛ N
N N
y = ( N 2 y2 + N3 y 3 + N 6 y6 ) = ⎨ y6 = ---------------
- = + -----6-⎞⎠ y 2 + ⎛⎝ N 3 + -----6-⎞⎠ y 3⎞⎠
ξ=1 ⎩ 2 ⎬⎭ ⎝⎝ 2 2 2 ξ=1

2 y3 – y2 2
⎧ ∂x x 3 – x 2 ∂y y 3 – y 2 ⎫ x 3 – x 2⎞
⎛ ---------------
⇒ ⎨ ------ = ----------------, ------ = ---------------- ⎬ ⇒ dS = h - + ⎛ ----------------⎞ dη = hLdη
⎩ ∂η ∂η
2 2 ⎭ ⎝ 2 ⎠ ⎝ 2 ⎠
1 1
1–η 1–η hL
f 2x = ∫
N 2 tx hLdη = ∫
N 2 ⎛⎝ ------------ t Ax + ------------ t Bx⎞⎠ hLdη = ------ t Ax .
2 2 3
–1 –1
hL hL hL
In the same way we obtain f2y = ------ t Ay , f 3x = ------ t Bx , f 3y = ------ t By ,
3 3 3
hL hL
f 6y = ------ ( 2t Ax + 2t Bx ) and f 6y = ------ ( 2t Ay + 2t By ) .
3 3

– 6.14 (24) –
FEM for Engineering Applications—Exercises with Solutions / August 2008 / J. Faleskog

6.10 cont.
In (c) and (d) we have that t Ax = t Bx = tx = σ 0 cos θ – τ 0 sin θ and
t Ay = t By = t y = σ 0 sin θ + τ 0 cos θ
(c) For a 4-node element this give f 2x = f3x = hL e t x and f 2y = f 3y = hL e ty , where
L e = L ⁄ 3 . The nodal force vector becomes (nodes 1 to 4, global node numbering)
T
after assembly: F s = hLe t x ty 2t x 2t y 2tx 2t y t x t y …
hL hL hL
(d) For a 8-node element this give f 2x = f 3x = --------e t x , f2y = f 3y = --------e t y , f6x = --------e 4t x
3 3 3
hLe L
and f 6y = -------- 4t y , where L e = --- . The nodal force vector becomes (nodes 1 to 7,
3 3
global node numbering) after assembly:
T hL
F s = --------e t x t y 4tx 4t y 2t x 2t y 4t x 4t y 2t x 2t y 4t x 4t y t x ty …
3

1 1
T ab N 1 0 N 2 0 N3 0 N4 0 0 .
6.11 f b = ∫ ∫
N fv h ------ dξdη where N = and f v =
4 0 N 1 0 N2 0 N3 0 N4 –ρg
– 1 –1

N1 0 0 1 1
T 0 N1 0 f ρ ghab (---------------------------------
1 – ξ ) ( 1 – η )- ρ ghab
N fv = = 1y ; f1y = – ---------------- ∫ ∫
dξdη = – ----------------
–ρg 4 4 4
–1 –1

T ρ ghab
⇒ fb = – ---------------- 0 1 0 1 0 1 0 1
4

– 6.15 (24) –
FEM for Engineering Applications—Exercises with Solutions / August 2008 / J. Faleskog

6.12

y/L Displacement boundary conditions: d1x = d1y = d2x = d2y = 0


give rise to the reaction forces: R1x, R1y, R2x, R2y
t = p0 1 – η
3
The external load acts on side x = 0, between nodes 1 and 3,
0
there, the shape functions reduces to
1 2 N 1 = 1 – η, N 3 = η, N 2 = 0
x/L
The consistent nodal force vector becomes (use subst. ξ = x/L, η = y/L):
1
T
fs = ∫ N ξ=0
thLdη ⇒ f 2x ≠ 0, f 3x ≠ 0 others equal to zero!
0 1 1
(----------------
1 – η )- σ 0 hL (-----------------
1 + η) σ 0 hL
f 1x = ∫
ηhLdη = – ------------- f 3x = ∫
ηhLdη = -------------
2 3 2 3
–1 –1
R1x + p 0 hL ⁄ 3 Reduced Eq. syst. (5) & (6):
R 1y
3Eh d p 0 hL 1
Load vector: F =
R 2x ---------- 1 0 3x = -----------
-
16 0 3 d 6 0
R 2y 3y

p 0 hL ⁄ 6
d 3x 8p
0 ⇒ = --- ----0- L 1
d 3y 9E 0
Reaction forces:
3Eh p 0 hL p0 hL 3Eh p 0 hL
Eq. (1): R1x = ---------- ( – d 3x ) – ------------ = – ------------ Eq. (2): R 1y = ---------- ( – d 3x ) = – -----------
-
16 3 2 16 6
3Eh p 0 hL
Eq. (3): R2x = 0 Eq. (4): R2y = ---------- ( d 3x ) = -----------
-
16 6

6.13(a) 2 x1 = 0 x2 = L ⁄ 2 x 3 = 0 the shape functions


L
With A e = -----
4 y1 = 0 y2 = L ⁄ 2 y 3 = L are obtained as
1
N 1 = --------- [ ( y 2 – y 3 ) ( x – x 2 ) + ( x 3 – x 2 ) ( y – y 2 ) ] = 1 – ξ – η
2Ae
1
N 2 = --------- [ ( y 3 – y 1 ) ( x – x 3 ) + ( x 1 – x 3 ) ( y – y 3 ) ] = 2ξ
2Ae
1
N 3 = --------- [ ( y 1 – y 2 ) ( x – x 1 ) + ( x 2 – x 1 ) ( y – y 1 ) ] = η – ξ
2Ae

– 6.16 (24) –
FEM for Engineering Applications—Exercises with Solutions / August 2008 / J. Faleskog

6.13(b) The stresses in the element are given by σ = CBde


∂N 1 ⁄ ∂x = – 1 ⁄ L ; ∂N 1 ⁄ ∂y = – 1 ⁄ L ⎫
–1 0 20
⎪ 1 1
∂N 2 ⁄ ∂x = 2 ⁄ L ; ∂N 2 ⁄ ∂y = 0 ⎬⇒ B = B 1 B 2 B 3 B 1 = --- 0 –1 B 2 = --- 0 0
⎪ L L
∂N 3 ⁄ ∂x = – 1 ⁄ L ; ∂N 3 ⁄ ∂y = 1 ⁄ L ⎭ –1 –1 02

–1 0
1
B 3 = --- 0 1
L
1 –1
⎧ P.S. ⎫ d1
3 1 0
C =
⎪ ⎪
=
3E
------
- d e = d 2 where d 1 = 0 d2 = 1⁄4 d = 0
⎨ 1--- ⎬
⎪ν = 3 ⎪ 8 1 3 0 0 – 1 ⁄ 108
3
–1 ⁄ 6
⎩ ⎭ 0 0 1 d3

σ xx d1 1⁄2
Thus σ = σ yy = C B 1 B 2 B 3 d 2 = C [ B 1 d1 + B 2 d 2 + B 3 d3 ] = Eε 0 0
σ xy d3 1 ⁄ 18

6.14 Stiffness 3 1 –2 –1 0 0 –1 0
matrix: 1 3 0 –1 0 0 –1 –2 –1
–2 0 3 0 –1 –1 0 1 Displacements are given by: D = K F
Eh – 1 –1 0 3 0 –2 1 0
K = -------
4 0 Stresses are given by: σ = CBD
0 –1 0 3 1 –2 –1
0 0 –1 –2 1 3 0 –1
–1 –1 0 1 –2 0 3 0
0 –2 1 0 –1 –1 0 3

(a) Displ. B.C. (symmetry & remove rigid body motion): D1x = D1y = D2y = D4x = 0
Force vector: T σ 0 hl σ 0 hl
F = F 1x F 1y F 2x F 2y F 3x F 3y F 4x F 4y = R R ----------
- R 2y ----------
- 0 R 4x 0
1x 1y 2 2

⇒D T σ0
= ------ l 0 0 1 0 1 0 0 0
E
T
el. 1 & el. 2: σ = σ 0 1 0 0 exact!

(b) Displ. B.C. (remove rigid body motion): D1x = D1y = D2y = 0

Force vector: F =
T
⎛⎝ R τ 0 hl ⎞⎠ ⎛⎝
τ 0 hl τ 0 hl ⎞⎠ ⎛⎝ ⎞⎠
τ 0 hl τ 0 hl τ 0 hl τ 0 hl τ 0 hl ⎛⎝ ⎞⎠
– ---------- R 1y – ---------- – ---------- R 2y + ---------- ---------- ---------- ---------- – ----------
1x2 2 2 2 2 2 2 2

⇒ T τ0
D = ---------- l 0 0 0 0 1 0 1 0
E⁄2
T
el. 1 & el. 2: σ = τ 0 0 0 1 exact!
Note! E/2 = G (shear modulus) in this case!

(c) Displ. B.C. (remove rigid body motion): D1x = D1y = D2y = 0
Force vector: T
F = R
1x R 1y –
ρghl - ⎛⎝
-------------
2
ρghl ⎞⎠ ⎛⎝ ρghl
2
⎞⎠ ρghl
2
0 R 2y – -------------- 0 – -------------- 0 – --------------
2

6 3 6 3

⇒D
2
T ρgl T ρgl T ρgl
= – "" ----------- 0 0 – 1 0 3 9 2 15 el. 1: σ = -------- 1 – 15 – 1 el. 2: σ = -------- – 1 – 9 1
24E 24 24

– 6.17 (24) –
FEM for Engineering Applications—Exercises with Solutions / August 2008 / J. Faleskog

6.15(a)
2L
T
Consistent nodal force vector: fs = ∫
N 2–3
th ds ⇒ f 2y ≠ 0, f3y ≠ 0
2L 0 2L
⎧ s ⎫ s -⎞ σ 0 σ 0 Lh
f2y = ∫
N 2 t y h ds = ⎨x = L – ------- ⎬ = ∫
--1- ⎛ L – ------ ------- h ds = ------------
-
⎩ 2⎭ L⎝ 2⎠ 2 2
0 0
2L 2L
⎧ s ⎫ σ0 σ 0 Lh
--1- ------
s - ------
f3y = ∫
N 3 t y h ds = ⎨y = ------- ⎬ = ∫
- h ds = ------------
-
⎩ 2⎭ L 2 2 2
0 0
Zero displacement B.C.: d1x=d1y=d2y=d3x=0 => reaction forces: R1x, R1y, R2y, R3x

Node force vector: F = R R 0 ⎛ R + σ 0 Lh⎞ σ 0 Lh


T
------------
- R 3x ------------
-
1x 1y ⎝ 2y 2 ⎠ 2
6.15(b)
Eq. (3) and (6) gives the reduced equation system
σ 0 Lh 0 σ
= L -----0- – 1 ⁄ 3
3Eh d d 2x
---------- 3 1 2x = ------------
- ⇒
16 1 3 d 2 1 d 3y E 1
3y
3Eh
Reaction forces: Eq. (1): R1x = ---------- ( – 3d 2x – d 3y ) = 0
16
3Eh σ 0 Lh
Eq. (2): R1y = ---------- ( – d 2x – 3d 3y ) = – -------------
16 2
σ 0 Lh σ 0 Lh
Eq. (4): R2y + ------------- = 0 ⇒ R2y = – ------------ -
2 2
Eq. (6): R3x = 0
6.15(c)

The strains in the element are given by: ε = Bd e


N1 = 1 – x ⁄ L – y ⁄ L ⇒ ∂N 1 ⁄ ∂x = –1 ⁄ L ; ∂N 1 ⁄ ∂y = – 1 ⁄ L ⎫ B = B1 B2 B3 =

N 2 = x ⁄ L ⇒ ∂N 2 ⁄ ∂x = 1 ⁄ L ; ∂N 2 ⁄ ∂y = 0 ⎬⇒
⎪ –1 0 1 0 0 0
N 3 = y ⁄ L ⇒ ∂N 3 ⁄ ∂x = 0 ; ∂N 3 ⁄ ∂y = 1 ⁄ L ⎭
1
= --- 0 – 1 0 0 0 1
L
–1 –1 0 1 1 0
0
εx 0
–1 0 1 0 0 0 σ σ –1 ⁄ 3 Agrees with the
= --- 0 –1 0 0 0 1 L -----0- – 1 ⁄ 3 = -----0- 1
1
ε = εy exact solution
L E 0 E
γxy – 1 –1 0 1 1 0 0 when ν = 1/3
0
1

6.16(a) Displacement boundary conditions: d1x=d3y=d4x=d4y=0

– 6.18 (24) –
FEM for Engineering Applications—Exercises with Solutions / August 2008 / J. Faleskog

6.16(b)
L
T
Consistent nodal force vector: fs = ∫
N 2–3
th ds ⇒ f 2x ≠ 0, f3x ≠ 0 others are
–L
equal to
L 1 zero!
– η-
1-----------
f 2x = ∫
N 2 tx h dy = ∫
σ 0 hL dη = σ 0 hL
2
–L –1
L 1
+ η-
1------------
f 3x = ∫
N 3 tx h dy = ∫
σ 0 hL dη = σ 0 hL
2
–L –1

Displacement boundary conditions give the reaction forces: R1x, R3y, R4x, R4y
T
Node force vector: Fe = R1x 0 σ 0 Lh 0 σ 0 Lh R 3y R 4x R 4y

6.16(c)
Force vector (external forces + reaction forces) can be calculated as Fe = K e D e
⇒ F1x = R 1x = – σ 0 hL ; F2x = σ 0 hL ; F3x = σ 0 hL ; F4x = R4x = – σ 0 hL
F 1y = 0 ; F2y = 0 ;F 3y = R3y = 0 ; F4y = R 4y = 0

6.17(a) Nodal force vector:


y
4 3 On the boundary x = 2L, between nodes 2 and 3,
t = σ0 y ⁄ L the shape functions take the values:
0 (1 – η) (1 + η)
x N 1 = N 4 = 0 och N 2 = -----------------, N 3 = -----------------
2 2
Consistent nodal force vector becomes (subst. η = y/L):
1 2 1
T
fe = ∫
N ξ=1
thLdη ⇒ f 2x ≠ 0, f 3x ≠ 0 others zero!
1 –1 1
(----------------
1 – η )- σ 0 hL (-----------------
1 + η) σ 0 hL
f 2x = ∫
ηhLdη = – ------------- f 3x = ∫
ηhLdη = -------------
2 3 2 3
–1 –1
Inclusive of reaction forces, the
F = R 0 –σ 0 hL σ 0 hL
T
nodal force vector becommes: 1x ------------
- 0 ------------
- 0 R4x R4y
3 3

– 6.19 (24) –
FEM for Engineering Applications—Exercises with Solutions / August 2008 / J. Faleskog

6.17(b) Stresses are calculated as: σ = Cε where ε = Bd e

Nodal force vector (given):


4σ 4σ
where d 1 = d 4 = 0 and d 2 = --- -----0- L – 1 , d3 = --- -----0- L 1
T
de = dT d T dT d T
1 2 3 4 3 E –1 3 E –1
B-matrix:
B = B1 B2 B3 B4 with Jacobi matrix J = ∂x ⁄ ∂ξ ∂y ⁄ ∂ξ = L 0
∂x ⁄ ∂η ∂y ⁄ ∂η 0 L

1–η 0 1+η 0
1 1
B 2 = ------
4L 0 –(1 + ξ ) , B 3 = ------
4L 0 1+ξ
Strains: –( 1 + ξ ) 1 – η 1+ξ 1+η

εx 1- 4--- σ
-----0- L [ – ( 1 – η ) + ( 1 + η ) ] 2--- σ
-----0- η
-----
4L 3 E 3E
ε = εy = B 2 d2 + B 3 d 3 = =
εy εy
γxy
γ xy γ xy
Stresses:
2 σ 2--- Note!
σx E 0 0 --- -----0- η σ η
3E 3 0 The solution based on one
σ = σy = Cε = 0 E 0 = FEM element deviates from the
εy σy
τ xy 0 0 E⁄2 exact solution:σ x = σ 0 y ⁄ L
γxy τ xy
σ y = τ xy = 0
plane stress (ν = 0.3)

– 6.20 (24) –
FEM for Engineering Applications—Exercises with Solutions / August 2008 / J. Faleskog

6.18 (a)
L ⎛ (L – x) ⎞
The contribution from the volume force is: F b =
T
N KdV = ⎜ N Khdy⎟⎟ dx
T
∫ ∫⎜ ∫
Ve 0 ⎝ –( L – x ) ⎠
Here, both f bix ≠ 0 and f bix ≠ 0 i = 1, 2, 3
L ⎛ (L – x) ⎞ L
2 2 3
⎜ 1--- ⎛ x y 2 ρω h 2 ρω hL
f b1x = ∫⎜ ∫
1 – --- – ---⎞ ρω xhdy⎟ dx = ------------- ∫ ( L – x ) xdx = -------------------
2 ⎝ L L ⎠ ⎟ L 12
0 ⎝ –( L – x ) ⎠ 0
L ⎛ (L – x) ⎞ L
2 2 3
⎜ x- 2 ⎟ dx ρω h 2 ρω hL
f b2x = ∫⎜ ∫
-- ρω xhdy⎟
= ------------- ∫ 2x ( L – x )dx = -------------------
L L 6
0 ⎝ –( L – x ) ⎠ 0
L ⎛ (L – x) ⎞ L
2 2 3
--x- + --y-⎞ ρω 2 xhdy⎟ dx = ρω
------------h- ∫ ( L – x ) 2 xdx = ρω
⎜ 1--- ⎛ hL
-------------------
f b3x = ∫⎜ ∫
1 –
2⎝ L L⎠ ⎟ L 12

0 –( L – x ) ⎠ 0

L ⎛ (L – x) ⎞ L
2
⎜ 1--- ⎛ x y ρ gh 2 ρ ghL
f b1y = ∫⎜ ∫
1 – --- – ---⎞ ( – ρ g )hdy⎟ dx = – ---------- ∫ ( L – x ) dx = – ----------------
2 ⎝ L L ⎠ ⎟ L 3
0 ⎝ –( L – x ) ⎠ 0
L ⎛ (L – x) ⎞ L
2
⎜ x ⎟ dx 2 ρ gh ρ ghL
f b2y = ∫⎜ ∫
--
- ( – ρ g )hdy

= – ------------- ∫ x ( L – x )dx = – ----------------
L L 3
0 ⎝ –( L – x ) ⎠ 0
L ⎛ (L – x) ⎞ L
2
⎜ 1--- ⎛ x y ρ gh ρ ghL
1 – --- + ---⎞⎠ ( – ρ g )hdy⎟⎟ dx = – ---------- ∫ ( L – x ) dx = – ----------------
2
f b3y = ∫⎜ ∫ 2 ⎝ L L L 3

0 –( L – x ) ⎠ 0
The displacement B.C:s : d1x = d1y = d3x = 0 give rise to reaction forces!
1 0 R1x
0 1 R1y
2 3 2
ρω hL ρghL
The total nodal force vector becomes: f = ------------------- 2 – ---------------- 0 + 0
12 0 3 1 0
1 0 R3x
0 1 0
6.18 (b)
Displacement boundary conditions: d1x = d1y = d3x = 0
Reduced equation system with 8 0 0 d 2x 2 3 2 2 0
------- 0 4 –2 d 2y = ρω
respect to the boundary: Eh hL - ρghL
------------------ 0 – ---------------
- 1
conditions, Eq. (3,4,6): 8 12 3
0 –2 3 d 3y 0 1

d 2x 2 3 1 2 0
ρω L ρ gL
⇒ d 2y = ---------------- 0 – – ------------ 5
6E 3E
d 3y 0 6

– 6.21 (24) –
FEM for Engineering Applications—Exercises with Solutions / August 2008 / J. Faleskog

6.18 (c) The stresses in the element is given by σ = Cε ,


where ε = Bd e = B 1 d 1 + B 2 d2 + B 3 d 3

Here we have 1 0 0 2 3 2 2
ρω L- 1 ρ gL 0 ρ gL 0
C = { ν = 0 } = E 0 1 0 ; d 1 = 0 ; d2 = --------------- – ------------ ; d3 = – ------------
6E 0 3E 5 3E 6
0 0 1⁄2
∂N 2 ⁄ ∂x 1 0 0 ∂N 3 ⁄ ∂x 0 –1 0
1 1-
B2 = 0 ∂N 2 ⁄ ∂y = --
- ; B = 0 ∂N 3 ⁄ ∂y = -----
L 0 0 3 2L 0 1
∂N 2 ⁄ ∂y ∂N 2 ⁄ ∂x 0 1 ∂N 3 ⁄ ∂y ∂N 3 ⁄ ∂x 0 –1
Thus
σx 1 0 0 ⎛
0 1 0 ⎛ 2 3 2 –1 0 ⎛ 2

⎜ 1- ρω L - 1 ρ gL 0 ⎞ 1 ρ gL ⎞⎟
σ = σy = E 0 1 0 ⎜ + --
0 L 0 0 ⎜ 6E --------------- – ------------ ⎟ + ------ 0 1 ⎜ – ------------ 0 ⎟⎟
1 ⎜ ⎝ 0 3E 5 ⎠ 2L ⎝ 3E 6 ⎠⎟
τ xy 0 0 --- ⎝ 0 0 1 0 –1 ⎠
2
σx 2 2 1 0
ρω L ρ gL
⇒σ = σy = ---------------- 0 – ---------- 3
6 3
τ xy 0 1
Note that the stress solution is approximate and rather poor in this case!
To obtain a better solution, many more elements are needed!

6.19(a)

Body force vector in element 2:


1 1
2 2
N K h J dξdη where K = ρω x = ρω L ( 3 + ξ )
T T
f b2 = ∫
N KdV = ∫ ∫
Ve –1 –1
0 0
2
1 1 J = L
(---------------------------------
1 – ξ ) ( 1 – η -) 2 2
⇒ f 1x = ∫ ∫
ρω L ( 3 + ξ )hL dξdη f 4x = f 1x
4
–1 –1
1 1
(----------------------------------
1 + ξ ) ( 1 – η )- 2 2
f 2x = ∫ ∫
ρω L ( 3 + ξ )hL dξdη f 3x = f 2x
4
–1 –1
2 3
T ρω hL
⇒ f b2 = ------------------- 8 0 10 0 10 0 8 0
3

– 6.22 (24) –
FEM for Engineering Applications—Exercises with Solutions / August 2008 / J. Faleskog

6.19(b) Stresses in the element is given by σ = Cε = CBd e

1 0 0 d1 = d4 = 0 2 3
Here, C = E 0 1 0 d T = d T d T d T d T where 44 ρω L
δx = ------ ----------------
1 2 3 4 e1 δ 3 E
0 0 1⁄2 d2 = d3 = x
0

⇒ σ = C ( B 1 d 1 + B 2 d 2 + B 3 d 3 + B 4 d 4 ) = C ( B 2 d2 + B 3 d 3 )

N i, ξ 0 1 0 1 0
1 1 1-
B i = --- 0 N i, η ⇒ B 2 = ------ 0 –( 1 + ξ ) ; B 3 = ----- 0 (1 + ξ)
L 4L 4L
N i, η N i, ξ –( 1 + ξ ) 1 (1 + ξ) 1

σ xx 1 0 0 δ x + δx Eδ x ⁄ ( 2L ) 1
1 22 2 2
⇒ σ yy = E 0 1 0 -----
- 0 = 0 = -----
- ρω L 0
4L 3
τ xy 0 0 1⁄2 – ( 1 + ξ )δx + ( 1 + ξ )δ x 0 0

Comparison with the exact solution: x=0 x=L x = 2L


FEM Exact
σ xx – σ xx
Relative eroor = ----------------------------------
Exact
× 100% – 8.3 % – 2.2 % 22.2 %
σ xx

6.20(a) Boundary conditions: D 1x = D 1y = D 2x = D 3x = D 3y = D 4y = D 7y = 0

6.20(b) For element e4 yields:

0 y x y x
Shape functions: N 1 = 2 – ---, N 2 = --- + --- – 2, N 3 = 1 – ---
t = x L L L L
3 2 –p 0 --- L
L T
e4 f e = ∫ N y = 2L thdx ⇒ f 2y ≠ 0, f 3y ≠ 0 others zero!
0
1 L L
x x hLp 0 x x hLp 0
f 2y = ∫ --- ⎛⎝ – p 0 ---⎞⎠ hdx = – ------------ f 3y = ∫⎝
⎛1 – ---⎞⎠ ⎛⎝ – p0 ---⎞⎠ hdx = – ------------
L L 3 L L 6
0 0

Assembly of global nodal force vector including the reaction forces gives:
F1x = R 1x ; F1y = R1y ; F2x = R2x ; F3x = R3x ; F4y = R4y ; F7y = R7y

F3y = – hLp 0 ⁄ 6 ; F 6y = – hLp 0 ⁄ 3

– 6.23 (24) –
FEM for Engineering Applications—Exercises with Solutions / August 2008 / J. Faleskog

6.20(c) Stress in element 1 is given by σ = CBde


Shape functions:
N1 = 1 – x ⁄ L – y ⁄ L ⇒ ∂N 1 ⁄ ∂x = – 1 ⁄ L ; ∂N 1 ⁄ ∂y = – 1 ⁄ L ⎫ B = B 1 B 2 B 3 =

N 2 = x ⁄ L ⇒ ∂N 2 ⁄ ∂x = 1 ⁄ L ; ∂N 2 ⁄ ∂y = 0 ⎬⇒
⎪ –1 0 1 0 0 0
N 3 = y ⁄ L ⇒ ∂N 3 ⁄ ∂x = 0 ; ∂N 3 ⁄ ∂y = 1 ⁄ L ⎭
1
= --- 0 – 1 0 0 0 1
L
1 ν 0 d1 –1 –1 0 1 1 0
E
C = { P.S. } = -------------2- ν 1 0 d e = d 2 d = 0 (R.V.); d 2 = D 4x ; d 3 = 0
1
1–ν (1 – ν) 0 D 2y
0 0 ---------------- d3
2
σ xx
D ⁄L E ( D 4x + νD 2y ) ⁄ L
Thus σ = σ yy = C [ B 1 d1 + B 2 d 2 + B 3 d3 ] = C 4x = -------------2-
D 2y ⁄ L 1 – ν ( D 2y + νD 4x ) ⁄ L
σ xy

6.21(a) Boundary conditions: D 1x = D 1y = D 2x = D 2y = D 3x = D 3y = D 4x = D 4y = D 5x = D 5y = 0

6.21(b) For one element yields:


1
4 3 0 T
t = fs = ∫
N ξ=1
thbd η ⇒ f 2y ≠ 0, f 3y ≠ 0 others are equal
1 2 –τ0
–1
to zero!
1 1 1 1

f2y = ∫N 2
ξ=1
thbd η = – τ 0 hb ∫ 1 – η )-
(----------------
2
d η = – τ 0 hb f 3y = ∫N 3
ξ=1 ∫
( 1 + η)
thbd η = – τ 0 hb ----------------- d η = – τ 0 hb
2
–1 –1 –1 –1
Assembly of consistent nodal forces from elements gives non-zero components
according to: F 26y = F 30y = – τ 0 hb och F27y = F 28y = F 29y = – 2 τ 0 hb

6.21(c) Stresses in element 1 is given by σ = CBde


D6y d1
D6x
4 3 d2 D 7x D 6x
D7y de = where d 1 = d4 = 0 (B.C.); d2 = ; d3 =
1 2 d3 D 7y D 6y
D7x
d4
⎧ ∂N
⎪ --------i
1 ∂N i
1 0 0 = --- --------
⎧ P.S. ⎫ ⎪ ∂x a ∂ξ
C = ⎨ ⎬ = E 0 1 0 ; B = B 1 B 2 B 3 B 4 where ⎨
⎩ν = 0 ⎭ ⎪ ∂N i 1 ∂N
0 0 1⁄2 ⎪ -------- = --- --------i

∂y b ∂η
B2 and B3 evaluated in the centroid of element 1 (ξ = η = 0) becomes:
1⁄a 0 1⁄a 0
1 1
B 2 = --- 0 –1 ⁄ b ; B 3 = --- 0 1⁄b
4 4
–1 ⁄ b 1 ⁄ a 1⁄b 1⁄a
Thus σ xx ( D 6x + D 7x ) ⁄ a
E
σ = σ yy = CB 2 d 2 + CB 3 d 3 = --- ( D 6y – D 7y ) ⁄ b
4
σ xy ( D 6x – D 7x ) ⁄ ( 2b ) + ( D 6y + D 7y ) ⁄ ( 2a )

– 6.24 (24) –
FEM for Engineering Applications—Exercises with Solutions / August 2008 / J. Faleskog

7. FEM: Heat conduction

7.1 The Figure to the right shows a one dimensional


model of a cooling fin. At the left boundary the T = 80°C a
o
temperature is constant at 80 C. Along the remain- b x=L
ing boundary, heat is lost to the surrounding air by
convection. Determine the cooling effect of the fin,
i.e. calculate the heat flow across its left boundary. Convection
x
Also determine the displacement in the fin due to T ∞ = 20°C
thermal expansion, where it can be assumed that Data:
the fin is undeformed at 20o C. Analyse the prob- a = 1 cm, b = 0.4 cm, L = 8 cm,
lem by FEM and use two linear elements. Carry out –5
the analysis in two steps: (a) calculate the tempera- E = 80 GPa, α = 1.4 ⋅ 10
ture distribution in the fin and the heat flow at x = 0, k = 3 W/cm/oC, h = 0.1 W/cm2/oC
and (b) calculate the displacement in the fin.
The material has elasticity modulus E, thermal expansion coefficient α, thermal conductivity k
and convection coefficient h. The material data and the geometry is shown in the figure.

7.2 A wall made of two layers of different material is shown in the


right hand figure. The temperature at the right side of the wall is
kept constant at 20o C. At the left side, a heat flux arises due to 1 2
convection, where the ambient temperature is equal to −5o C. The
thermal convection coefficient is h and the thermal conductivity of
the two materials is k1 and k2, respectively. Determine the tempera-
ture distribution through the wall by FEM-analysis. In the present
case it suffice to model each layer by one linear element. L1 L2

Data: k1 = 0.2 W/cm/oC, k2 = 0.06 W/cm/oC, h = 0.1 W/cm2/oC, L1 = 2 cm and L2 = 5 cm.

7.3 A bar of copper is clamped between two rigid


walls. At point B, the bar is influenced by a point force T ∞ = 20°C
b
P0 = 2kN and a heat source keeping the temperature Konvektion
A b C
constant at 100o C. The right boundary (point C) is B P0
insulated and at the left boundary (point A) the tem-
perature is kept constant at 20o C. Between the end
points of the bar, a heat flux occurs by convection, L 2L
where the ambient temperature of the surrounding
medium is equal to 20o C. Conduct a FEM-analysis to determine the distribution of tempera-
ture, displacement and normal stress in the bar. Use two linear elements in each of the inter-
vals A-B and B-C, respectively. Carry out the analysis in the two consecutive steps: (a)
calculate the temperature and (b) calculate the displacement and normal stress.
Data: L = 10 cm, b = 1 cm, k = 3.9 W/cm/oC, h = 0.01 W/cm2/oC, E = 125 GPa and
–5
α = 1.8 ⋅ 10 .

– 7.1 (3) –
FEM for Engineering Applications—Exercises with Solutions / August 2008 / J. Faleskog

Solutions
7.1
(a) Temperature distribution Boundary conditions:
FEM-analysis: 2 linear elements x = 0: T = 80 oC
∂T
x = L: – kA ------ = hA ( T – T∞ )
T1 T2 T3 ∂x

T 1 = 80°C “Reaction
0.6733 –0.1133 0 11.20 + Q R heat flow”
Equation
– 0.1133 1.3467 – 0.1133 T2 = 22.40
system
0 –0.1133 0.7133 T3 12.00
unit [W/oC] unit [W]

⇒ T2
T3
= 25.12 °C
20.81
⇒Q R = 39.82 W

(b) Displacement
FEM-analysis: 2 linear elements Boundary conditions:
x = 0: u = 0
D1 D2 D3 x = L: σA = 0

1 –1 0 D1 = 0
3
– 1.459 ⋅ 10 + R ⎧⎪ D
⇒⎨ D = 0.01823 10 m
2 –3
6
80 ⋅ 10 – 1 2 – 1
⎪⎩
D2 = 1.326 ⋅ 10
3 0.01989
3
0 –1 1 D3 3
0.133 ⋅ 10 R = 0
unit [N/m]
unit [N]
Comparison between the exact solution and FEM-solutions based on 2, 4 and 8 elements
80

70 20
Exact solution
2 element
60 4 element
Displacement / μm
Temperature / C

8 element 15
o

50

40 10
Exact solution
30 2 element
5 4 element
20 8 element

10 0
0 2 4 6 8 0 2 4 6 8
x / cm x / cm

– 7.2 (3) –
FEM for Engineering Applications—Exercises with Solutions / August 2008 / J. Faleskog

7.2
FEM-analysis: 2 linear elements
l1 l2 ∂T
Boundary conditions, x = 0: –kA ------ = hA ( T – T∞ )
∂x
T1 T2 T3 x = L: T = -5 oC
k1 k
Equation
system
----- + h
l1
– ----1-
l1
0 ⎧⎪
⇒ ⎪⎨
T1 hT∞ T1 – 2.58 °C
k1 k1 k2 k2 =

⎪⎪
A – ----- ----- + ----- – ----- T2 = A 0 T2 – 0.161
l1 l 1 l 2 l2
Q ⁄A
T 3 = 20°C
⎩Q
2
k2 k2
R
R ⁄ A = 0.242 [ W ⁄ cm ]
0 – ----- -----
l2 l2
7.3

FEM-analysis: 4 linear elements Prescribed values:


T1 T2 T3 T4 T5 x = xA: T = 20o C
e1 e2 e3 e4
x = xB: T = 100o C
∂T
D1 D2 D3 D4 D5 x = xC: Q = – kA ------ = 0
∂x

(a) Temperature distribution, divide into two separate analysis, since T3 is prescribed.

T1 T2 T3 T3 T4 T5

0.8467 – 0.7467 0 T 1 = 20°C 2 + Q R1 0.5233 – 0.3233 0 T 3 = 100°C 4 + Q R3


– 0.7467 1.6933 – 0.7467 T2 = 4 – 0.3233 1.0467 – 0.3233 T4 = 8
0 – 0.7467 0.8467 T 3 = 100°C 2 + Q R3 0 – 0.3233 0.5233 T5 4

⇒T 2 = 55.27°C ⇒ T4
= 50.54 °C
T5 38.87

(b) Displacement and stress calculations


EA ΔT i = average temperature
Valid for each element: k e = ------- 1 – 1 ; f T = EAαΔT i – 1
li –1 1 1 change in element i
D1 = 0 – ΔT 4
2 –2 0 0 0 R1 – 3.968 R1
D2 0 ΔT 1 – ΔT 2
–2 4 –2 0 0 – 9.00 0
6 3
125 ⋅ 10 0 –2 3 –1 0 D3 = P 0 + EAα ΔT – ΔT = 2.532 10 + 0
2 3
0 0 –1 2 –1 D4 0 6.877 0
ΔT 3 – ΔT 4
0 0 0 –1 1 D =0 R5 5.559 R5
5 ΔT 4
D2
⇒ D3 =
– 14.08
–6
7.84 10 m
⇒ R1
R5
= 7487 N
– 9487
Stress in the elements: D4 31.43
D 2 – D1 D 3 – D2
σ 1 = E ------------------- – αΔT 1 = – 74.9 MPa, σ 2 = E ------------------- – αΔT 2 = – 74.9 MPa
l1 l2
D 4 – D3 D5 – D 4
σ 3 = E ------------------- – αΔT 3 = – 94.9 MPa, σ 4 = E ------------------- – αΔT 4 = – 94.9 MPa
l3 l4

– 7.3 (3) –

You might also like